Download ID_4093_Pulmonology (Current practice _English_sem_12

Survey
yes no Was this document useful for you?
   Thank you for your participation!

* Your assessment is very important for improving the workof artificial intelligence, which forms the content of this project

Document related concepts

Infection control wikipedia , lookup

Medical ethics wikipedia , lookup

Syndemic wikipedia , lookup

Dental emergency wikipedia , lookup

Prenatal testing wikipedia , lookup

Patient safety wikipedia , lookup

Adherence (medicine) wikipedia , lookup

Electronic prescribing wikipedia , lookup

Dysprosody wikipedia , lookup

List of medical mnemonics wikipedia , lookup

Transcript
Назва наукового напрямку (модуля):
Семестр: 12
Pulmonology (Current practice in internal medicine)
Опис:
situational tasks
Перелік питань:
1.
A. *
B.
C.
D.
E.
2.
A. *
B.
C.
D.
E.
3.
A. *
B.
C.
D.
E.
4.
A. *
B.
C.
D.
E.
5.
A. *
B.
C.
D.
E.
One worker in the surgical center is found to be culture-positive for streptococcal microorganisms in
his nasal mucosa but has no symptoms of infection. What is this patient's status in the chain of
infection?
Reservoir
Portal of entry
Portal of exit
Susceptible host
Non of above
20-years-old man suffers from bronchial asthma, the attacks of dyspnea arise 3-4 times a day. Night
attacks are present 2 times a week. FEV1 - 70 %, its variability during day is 30 %. What is your
diagnosis?
Moderate persistent BA
Status asthmaticus
Intermittent BA
Severe persistent BA
Mild persistent BA
47-year-old man is evaluated because of cough that has persisted for 6 months. He has no postnasal
drip, wheeze, or heartburn. His physical examination, chest radiograph, and spirometry are normal.
He receives no benefit from a 3-month trial of twice-daily proton-pump inhibitors, intranasal
corticosteroids, and antihistamines. He has a family history of allergies. Which of the following
would suggest the diagnosis of this patient?
Methacholine challenge testing
24-hour esophageal pH monitoring
CT scan of the sinuses
Bronchoscopy
CT scan of the chest
60-yr-old male non-smoker demonstrates such results of his pulmonary function tests: FEV1—1.4 l,
FVC—3.5 l, FEV1/FVC—40%. After bronchodilator trial—FEV1/FVC—59%. After 2 weeks of
prednisolone 30 mg daily FEV1/FVC = 72%. What is the diagnosis?
Bronchial asthma
COPD
Emphysema
Tracheal compression
Pulmonary fibrosis
A 10-yr-old boy presents with wheezing attacks and episodic shortness of breath. His PEFR is 400
l/min. What is the best method of treatment?
?2 agonist
Erythromycin
Plasmaphoresis
Cyclophosphamide
Co-trimoxazole
6.
A. *
B.
C.
D.
E.
7.
A. *
B.
C.
D.
E.
8.
A. *
B.
C.
D.
E.
9.
A. *
B.
C.
D.
10.
A. *
B.
C.
D.
E.
A 17-year-old previously healthy man presents with a history of shortness of breath on exertion,
particularly during basketball season, when he sometimes needs to sit down during practice to catch
his breath. He does not notice any shortness of breath with routine activity. There is no family history
of asthma. On physical examination, he is in no respiratory distress. His lungs are clear, with no
wheezing during either tidal breathing or forced expiration. His heart is normal. Baseline spirometry
is normal. What is the next diagnostic step?
Exercise testing with postexercise spirometry
Allergy testing
Methacholine challenge testing
Overnight oximetry
Repeat lung volumes and diffusing capacity
A 20-yr-old men presents with wheezing and shortness of breath. His PEFR is 400 l/min.
Spirography: FEV1, FVC and Typhno index are low. What is the diagnosis?
Asthma
COPD
Emphysema
Pulmonary embolism
Pneumonia
A 20-yr-old woman is too breathless to speak. Her pulse is 120/min, respiratory rate is 30/min and
PEFR is 100 l/min. Examination reveals a very quiet chest and CXR is normal. Choose the single
most likely management:
Nebulised salbutamol
Pleural aspiration
Heparin
Intramuscular adrenaline
Rapid infusion of saline
A 25-year-old man is evaluated because of a 3-month history of episodic dyspnea at rest. The
episodes occur approximately three times per week and are accompanied by cough. His symptoms
awaken him at night approximately three times per month. He had asthma as a child, which resolved.
His temperature is 36.5 C, pulse rate 85/min and regular, respiration rate 14/min, and blood pressure
125/75 mm Hg. The only abnormality noted on physical examination is bilateral wheezing without
crackles. Chest radiograph is normal. Spirometry shows a forced expiratory volume in 1 sec (FEV1)
78% of predicted, and a forced vital capacity 93% of predicted. He has a 17% (430 ml) improvement
in FEV1 after using bronchodilators. Which of the following is the most appropriate treatment
Albuterol
and
a low-dose
regimen for
this
patient? inhaled corticosteroid
Albuterol
Albuterol and a long-acting ?2-agonist
A long-acting ?2-agonist
A 28 years old patient, complaints on cough with small amount of colourless sputum, pain in the
right half of thorax during breathing, shortness of breath, increase of temperature to 39 °С. Felt ill
rapidly. Used aspirin. Objectively: herpes on lips. In lower lobe of right lung there is dull percussion
sound, bronchial breathing. X-ray: there is homogeneous infiltration of right lower lobe. What is the
most possible etiology of pneumonia?
Pneumococcus
Staphylococcus
Mycoplasma
Legionella
Klebsiella
11.
A. *
B.
C.
D.
E.
12.
A. *
B.
C.
D.
E.
13.
A. *
B.
C.
D.
E.
14.
A. *
B.
C.
D.
A 34-year-old woman is evaluated because of a 1-year history of increased dyspnea on exertion. She
has no symptoms at rest but has to stop about 15 minutes into her aerobics class because of dyspnea
and occasional cough. She usually recovers fully in about an hour. One year ago she was able to do
aerobics for 45 minutes without difficulty. Her vital signs are normal, and her physical examination is
normal, including clear breath sounds. Baseline spirometry is also normal. Which of the following
would be best next step in the management of this patient?
Inhaled albuterol prior to exercise
Oral leukotriene inhibitors
Long-acting theophylline
Inhaled ipratropium bromide prior to exercise
Inhaled corticosteroids
A 37-year-old man with asthma is evaluated because he continues to have frequent attacks and now
feels his short-acting ?2-agonist is not providing relief. He states he is using his medications,
including a long-acting ?2-agonist inhaler, inhaled high-dose corticosteroids, and a short-acting
?2-agonist inhaler as rescue medication. He has symptoms daily and nocturnal symptoms about twice
per week. On physical examination, he is in mild respiratory distress. He is afebrile. Pulse rate is
90/min and regular, respiration rate is 18/min, and blood pressure is 140/85 mm Hg. He has bilateral
wheezing. Spirometry shows a forced expiratory volume in 1 sec (FEV1) 65% of predicted; it
improves with bronchodilators to 85% of predicted. He has no history of recent viral upper
respiratory infections or rhinitis or symptoms of gastroesophageal reflux disease. Which of the
following is the best next step in this patient’s management?
Observe the patient using the metered-dose inhaler
Add a leukotriene inhibitor
Switch to an oral ?2-agonist and have the patient return for a pill count
Initiate oral prednisone therapy and have the patient return for a pill count
Have the patient return with a symptom and treatment log.
A 38 years old patient, who drunk a lot of alcohol, has severe pneumonia. His condition got worse,
the temperature of body rose to 39-40 °С, an unpleasant smell appeared from a mouth, increased
amount of purulent sputum; increased ESR and amount of band leucocytes. On the X-ray - in the
lower lobe of right lung there is massive infiltration with light area in a center. What complication is
it necessary to suspect?
Acute pulmonary abscess
Bronchiectasis
Infarction-pneumonia
Gangrene of lungs
Empyema of pleura
A 40-year-old woman, a nurse, is evaluated because of worsening asthma symptoms. She has had
mild, intermittent asthma since college, for which she has been using an albuterol inhaler as needed,
usually less than once a month. During the past 3 months, she has experienced cough, tightness of the
chest, and wheezing, which improve after the use of inhaled albuterol. She uses the inhaler twice a
day on average and has awakened at least twice a week with nocturnal cough. She works three
consecutive 12-hour day shifts, and the cough is regularly worse at the end of each shift. During her
days off, she has fewer asthma symptoms and feels significantly better by the time she returns to
work. She has a history of allergic rhinitis that has also recently become more symptomatic.
Approximately 6 months ago, she acquired a kitten that sleeps in the bedroom. She has lived in her
home for 6 years, and it is carpeted and has heavy draperies. Chest examination is notable for good
air entry. There are scattered end-expiratory wheezes. In addition to treatment with inhaled
corticosteroids, which of the following interventions is most likely to benefit this patient?
Avoiding exposure to latex products
Treatment with an oral antihistamine
Getting rid of the kitten
Removing the carpets and draperies from her home
E.
15.
A. *
B.
C.
D.
E.
16.
A. *
B.
C.
D.
E.
17.
A. *
B.
C.
D.
E.
18.
A. *
B.
C.
D.
19.
A.
B. *
Treatment with a leukotriene-modifying drug
A 46-year-old woman who works as a nurse is evaluated because of a 2-year history of episodic
wheezing and a squeaky voice. This past spring, her symptoms worsened, requiring her to seek
medical attention; she was placed on a short-acting ?2-agonist that did not provide much relief. She
has no history of wheezing and says that these changes began after a severe influenza infection 3
years ago. Currently she feels well and has had no symptoms for several months; she is not taking
any medications. Physical examination shows no abnormalities, and baseline spirometry is normal.
What is the best test to evaluate this patient’s condition?
Methacholine challenge testing
Bronchoscopy to evaluate her trachea
Exercise echocardiogram
CT scan of the sinuses
Non of above
A 48 years old patient, complaints on weakness, dyspnea, pain in the left half of thorax, permanent
cough with viscid sputum, in which particles of blood are sometimes determined. For the last 3
months lost 5 kg of body mass. On the X-ray of lungs there is total homogeneous shade determined
from the left side. Organs of mediastinum are displaced to the left. What diagnosis is possible?
Lung athelectasis
Lung gangrene
Total exudative pleurisy
Pneumonia
Empyema of pleura
A 53-year-old woman with a history of mild persistent asthma is evaluated because of a recent
increase in her symptoms, with dyspnea and cough occurring daily and a cough that awakens her
once a week. She is currently using low-dose inhaled corticosteroids. She has no symptoms of rhinitis
or gastroesophageal reflux. On physical examination, she has intermittent wheezing bilaterally.
Which of the following is the most appropriate change in her therapy?
Add a long-acting ?2-agonist
Initiate azithromycin therapy
Add a nebulized short-acting ?2-agonist
Add inhaled ipratropium bromide
Add a leukotriene inhibitor
A 55-year-old man is evaluated in the emergency department because of an acute, severe asthma
attack; he is hospitalized in the intensive care unit for aggressive medical therapy and monitoring. He
is expectorating thick greenish sputum. His medical history includes hypertension, cholecystectomy
and glaucoma. Chest radiograph reveals hyperinflation only. Medical therapy in the emergency
department included repeated doses of aerosolized albuterol and ipratropium, as well as
methylprednisolone, 125mg administered intravenously. Peak expiratory flow rate is unimproved at
80 l/min. Which of the following is the most appropriate next step in this patient’s management?
Intravenous magnesium sulfate
Nebulized ipratropium bromide administered by face mask
Broad-spectrum antibiotics targeting community-acquired respiratory pathogens
Inhaled corticosteroids
A patient who is 2 days postoperative from a bowel resection tells her physician that she is having a
hard time “catching her breath,” feels nauseated, and has chest pains when she inhales. The physician
suspects that she is having a pulmonary embolism. What intervention should the physician perform
before notifying the physician?
Increase the IV flow rate
Apply oxygen by mask or nasal cannula at 5 l/min
C.
D.
E.
20.
A. *
B.
C.
D.
E.
21.
A. *
B.
C.
D.
E.
22.
A. *
B.
C.
D.
E.
23.
A. *
B.
C.
D.
E.
24.
A. *
B.
C.
D.
E.
Assess the chest and axillary area for the presence of petechiae
Place the patient in shock position, with her head and neck flat and her legs elevated
Non of above
A young woman complains of wheeze, dyspnoea and cough. She cannot sleep at night because of a
chronic cough. She and her mother love animals and together they have 14 cats. Her PEFR is normal
but her CXR suggests hyperinflation. What is the previous diagnosis?
Bronchial asthma
Bronchogenic carcinoma
Emphysema
Respiratory failure
Bronchitis
After emotional exertion patient of 24 y.o. developed condition with dyspnea and prolonged
expiration, distantional wheezes, frequent night symptoms. All that limited his physical activity. PEV
and FEV1 < 60 %, daily variability -30 %. What diagnosis is possible?
Severe persistent bronchial asthma
Mild persistent bronchial asthma
Persistent bronchial asthma of moderate severity
Intermittent bronchial asthma
Bronchospastic syndrome of allergic origin
Drug abuser, a 41-yr-old man, presents with fever, cough and breathlessness. This was preceded by
viral influenza. Chest radiograph shows multiple abscesses. What is the most possible etiology of
disease?
Staphylococcus
aureus
Cryptococcus
Streptococcus pneumoniae
Legionella pneumonia
Mycobacterium avium
Female B., 44 years old, complains on cough with mucous sputum, increase of temperature to 39 °С,
weakness, dyspnea, sweating. Breathing rate - 26/min, skin is moist. Below left scapula there is
shortening of percussion sound. Breathing during auscultation is weakened, moist rales. Blood test: L
- 11х109/l, ESR - 29 mm/h. Your previous diagnosis?
Left-side lower lobe pneumonia
Gangrene of lungs
Left-side exudative pleurisy
Cancer of left side lower lobe
Pulmonary abscess
Female patient K., 46 years old, after decreasing of fever after flue noticed pain appeared in a thorax,
cough with yellow-green sputum (amount-150 ml a day), sometimes with some blood. Objectively:
breathing rate - 36/min. In lungs from the right side lower scapula there is dull sound during
percussion, hard breathing, and moist rales. Blood test: L - 18,6х109/l, ESR -64 mm/h. Analysis of
sputum: L -80-100 , Er - 40-50, elastic fibers, cocci. X-ray: lung roots are enlarged, from the right
side lower lobe is heterogeneously infiltrated with two lighter areas. What is the most possible
previous
Right-sidediagnosis?
pneumonia with abscesses
Peripheral cancer
Infiltrative tuberculosis in the phase of disintegration
Exudative pleurisy
Infarction-pneumonia
25.
A. *
B.
C.
D.
E.
26.
A. *
B.
C.
D.
E.
27.
A. *
B.
C.
D.
E.
28.
A. *
B.
C.
D.
E.
29.
A. *
B.
C.
D.
Female, 34 years old, has an increase of body temperature to 38 °С, cough with purulent sputum,
weakness, dyspnea, pain in a thorax during breathing. During percussion there is shortening of sound
in the lower part of left lung, during auscultation – moist rales. What method of investigation is the
decisive one to confirm diagnosis?
X-ray examination
Bacteriological analysis of sputum
Spirometry
Pneumotachometry
Bronchography
Girl, 18 y.o., pets seller, complaints mainly during working time on the attacks of dry cough, feeling
of running nose. She often ills with viral respiratory infections. Her mother is ill with bronchial
asthma. Objectively: breathing rate - 18/min. Heart rate - 80/min, BP - 110/70. In lungs vesicular
breathing, dry wheezes are heard in distance. Tones of heart are weaker than normally. Test with
berotec showed reversibility of bronchial obstruction. What tactic will be the best for the patient?
To change job
To use intal
To use monteleucast
To use berotec constantly
To use antihystaminic preparations
Girl, 23 y.o., for 2 years is ill with bronchial asthma. Recently attacks of dyspnea became more
frequent and started to arise 4-5 times a week, night attacks - 2-3 times a month. She used salbutamol
to remove the symptoms. Test with the antigen of home dust is positive. Objectively: condition is
satisfactory. Breathing rate - 20/min. Heart rate - 76/min, BP -120/80. In lungs breathing is vesicular.
Tones of heart are a little weak, rhythm is normal. What mechanism is desicive in development of
bronchial obstruction in this case?
Hyperreactivity of bronchi
Тrachео-bronchial dyskinesia
Violation of metabolism of arachidonic acid
Adrenergic disorders
Activity of the parasympathetic nervous system is increased
Male patient F., 48 years old, during a week stayed at home with diagnosis of respiratory viral
infection. Doctor noticed complaints on cough with small amount of mucus-purulent sputum,
weakness. Objectively: condition is relatively satisfactory. T - 37,2 °С. Breathing rate - 18/min.,
pulse - 80/min., BP - 110/70. In lungs there is vesicular breathing, with a hard tint, single dry
wheezes. Tones of heart are a little dull, rhythm is correct. What is the treatment tactic?
To prescribe antibacterial therapy
To stay at home for some more days
To go to work
To send patient to pulmonologist
To hospitalize patient to the pulmonological department
Male patient G., 56 years old, complaints on permanent pain in a thorax which disturbs for last 2
months. Pain is not connected with breathing. There is also cough with particles of blood in sputum.
Weakness, fatigue are present. On the chest X-ray in the lower lobe of right lung there is spherical
shadow, size 4x6 cm, related to the lungs rhadicis. What is the most possible diagnosis?
Perypheral lung cancer
Tuberculoma
Metastasis
Pulmonary abscess
E.
30.
A. *
B.
C.
D.
E.
31.
A. *
B.
C.
D.
E.
32.
A. *
B.
C.
D.
E.
33.
A. *
B.
C.
D.
E.
34.
A. *
B.
C.
D.
E.
Pneumonia
Man, 32 y.o., complaints on attack of expiratory dyspnea, which lasts for 48 hours, cough with small
amount of sputum. He is ill with bronchial asthma for 5 years, was treated with glucocorticosteroids,
used inhalers. Objectively: condition is severe, patient sits. Diffuse cyanosis, pulse -110/min, BP 110/70. Tones of heart are weak, II tone is louder above the pulmonary artery. During percussion in
lungs there is “bang-box” sound, large amount of dry wheezes. In blood there is eosinophylia - 18 %.
What medicines are drugs of choice for this patient?
Corticosteroids
?2-agonists
Theophyllin
Cholynolytics
Antihystamines
Man, 39 y.o., 8 last years is ill with bronchial asthma. Rapidly during physical work he felt
worsening of breathing, cough, distance wheezes appeared and dyspnea began to increase. Medicine
of what pharmacological group is it better to recommend for the patient to remove such attacks of
dyspnea? of ?2-adrenoreceptors
Agonists
Metilxantines
?2-adrenoblockers
Inhalated glucocorticoids
Oral glucocorticoids
Man, 46 y.o., suffers for the last 10 years from bronchial asthma. Rapidly during physical work he
felt worsening of breathing, cough, distance wheezes appeared and dyspnea began to increase.
Medicine of what pharmacological group is it better to recommend for the patient to remove such
attacks of dyspnea?
Agonists of ?2-adrenoreceptors
Atropine
Intal
Epinephrine
Monteleucast
Man, 43 y.o., complaints on dyspnea during physical activity. Objectively: temperature 36,4 °С,
breathing rate - 20/min, pulse - 78/min, BP-125/80. Emphysematous form of thorax. In lungs – weak
vesicular breathing. What test should be passed by patient at home to decide question about
efficiency of prescribed broncholytics?
Peakflowmetry
Spirography
ECG-control of overload of right chambers of heart
Bronchoscopy
Analysis of sputum (amount and microscopy)
Patient, 28 y.o., has running nose, attacks of dyspnea at night once a week. Felt ill after viral
respiratory infection which was treated with acetilsalicylic acid. Eosynophylia was founded in blood
and sputum. What disease may be suspected?
Aspirin bronchial asthma
Eosinophylic infiltrate of lungs
Bronchial asthma of physical exertion
Allergic rhinitis
Bronchial asthma, exogenous form
35.
A. *
B.
C.
D.
E.
36.
A. *
B.
C.
D.
E.
37.
A. *
B.
C.
D.
E.
38.
A. *
B.
C.
D.
E.
39.
A. *
B.
C.
D.
E.
Patient, 30 y.o., after a viral infection has daily symptoms of dyspnea, which causes lowering of
activity and bad sleep; night symptoms are more frequent then once a week. PEV and FEV1 - 60-80
%, day variability > 30 %. There is a necessity of daily usage of ?2-agonists of short action. What is
the diagnosis?
Persistent bronchial asthma of moderate severity
Mild persistent bronchial asthma
Intermittent bronchial asthma
Severe persistent bronchial asthma
Status asthmaticus
Patient, 42 years old, complaints on attacks of dyspnea, every time uses 1-2 doses of salbutamol.
These attacks are accompanied with cough and minimal amount of viscid glassy sputum. He is ill for
8 years. Objectively: temperature - 36,7 C; breathing rate – 21/min.; pulse-90/min.; BP - 130/80;
FEV1 - 77 %. In lungs – solitary dry wheezes. Blood test: eosinophyles - 6 %. What medicines are
“basic” in the treatment of this patient?
Antiinflammatory
Cholynolytics
Mucolytics
Antihystaminic
?2-agonists
Patient, 44 y.o., complaints on attack of dyspnea, which arises suddenly at night. Connects this attack
with overcooling. He is ill for more than 10 years. Thorax of emphysematous form. During
percussion in lungs – “bang-box” sound. During auscultation there is plenty of dry wheezes. In
blood: moderate leucocytosis, eosinophylia - 10 %. On the chest X-ray film – increased
pneumatization of pulmonary tissue. What diagnosis is the most possible?
Bronchial asthma, exacerbation phase
Bronchiectasis, exacerbation phase
COPD, exacerbation phase
Chronic bronchitis
Eosinophylic pulmonary vasculitis
Patient J., 45 y.o., complaints on dyspnea during insignificant physical exertion, cough with minimal
amount of “glass-like” sputum, attacks of dyspnea up to 3 times a day, more often at night, sweating.
She is ill for more than 5 years. Has an allergy on dust, cockroaches. For treatment uses bekotid for
near the year. Diagnosis?
Bronchial asthma
Eosinophylic pulmonary infiltrate
COPD
Bronchiectasis with bronchial spasm
Pulmonary vasculitis (syndrome of Charg - Stross)
Patient G., 47 y.o., with long history of bronchial asthma, has developed more frequent attacks of
dyspnea. Inhalations of astmopent and berotec are not effective. From prescription of what medicine
is it better to begin the intensive treatment?
Glucocorticoids
Oxygen therapy
Bronchodylators
Infusion therapy
Heart glycosides
40.
A. *
B.
C.
D.
E.
41.
A. *
B.
C.
D.
E.
42.
A.
B.
C.
D. *
E.
43.
A. *
B.
C.
D.
E.
44.
A. *
B.
C.
D.
E.
45.
A.
B.
Patient Y.,49 y.o., complaints on dyspnea, cough. Sputum is absent. Used many puffs of salbutamol,
intal, but without any efficacy. Objectively: sits, leaning on a table. Total cyanosis of the body.
Peripheral edema is absent. Breathing is superficial, dyspnea, during auscultation breathing cannot be
heard in some areas of lungs; wheezes are diffuse, expiration is considerably prolonged. Tones of
heart are weak, tachycardia. Pulse - 112/min, BP - 110/70. Liver is near the edge of costal arch. What
is the previous diagnosis?
Status asthmaticus
Bronchial asthma of moderate severity
COPD
Aspiration of foreign body
Heart asthma
Patient A., 35 y.o., noticed infrequent (rarer than 1 time a week) attacks of dyspnea, which are easily
removed with inhalations of ?2-agonists of short action. During attack in lungs are heard dry
wheezes, between attacks FEV1 is more than 80 % from normal. What is the diagnosis?
Intermittent bronchial asthma
Persistent bronchial asthma of moderate severity
Mild persistent bronchial asthma
Severe persistent bronchial asthma
Given information is not enough for determination of severity of bronchial asthma
Patient B., 25 years old engineer, appeared during a fire in the area of high concentration of CO (an
industrial accident). In hospital delivered in the irresponsible state. What laboratory tests are the early
criteria of estimation of severety of the state?
Estimation of blood viscosity
Anemia
Leucocytosis
Carboxihemoglobinemia
Methemoglobinemia
Patient complaints on attacks of dyspnea, which arise 1-2 times a week, night symptoms - 2 times a
month and even more frequently. For a patient night sleep is violated as a result of attacks of
dyspnea. FEV1 > 80 % from normal. What diagnosis would you suspect?
Mild persistent BA
Severe persistent BA
Intermittent BA
Moderate persistent BA
Status asthmaticus
Patient E., 43 years old, worker of coal mine, complaints on expiratory dyspnea, cough with dark
sputum. On roentgenogram there are the linear-reticulated diffuse changes. Doctor thinks about
anthracosis. Which characteristic is peculiar for the coal dust pneumoconiosis?
Raised activity of phagocytosis
Stimulation of carcinogenesis
To cause the unspecific allergic reactions
To cause the considerable mechanical irritation
To form colloid precipitates
Patient F., 46 years old, was hospitalized urgently with acute attack of dyspnea. Last 5 years he has
been working on poultry farm. During examination bronchial asthma was diagnosed. What additional
diagnostic methods are necessary to confirm the professional genesis of asthma?
sanitary-hygienic characteristics of the work conditions
echocardioscopy
C. *
D.
E.
46.
A.
B.
C.
D.
E. *
47.
A. *
B.
C.
D.
E.
48.
A. *
B.
C.
D.
E.
49.
A. *
B.
C.
D.
allergic and immunological tests
investigation of the function of external breath
roentgenography of pulmonary system
Patient G., 36 years old, works on a poultry factory. She was emergently hospitalized with acute
attack of dyspnea. During observation bronchial asthma was diagnosed. What additional methods of
research must be conducted above all things to confirm the professional genesis of bronchial asthma?
roentgenologic research of breathing organs
professional route of patient
sanitary-hygienic characteristic of work conditions
research of external breathing function
allergic and immunological tests
Patient H., a 52 years old man, is evaluated because of a 2-month history of nonproductive cough,
myalgias, and low-grade fever. When his illness began, a chest radiograph showed bilateral alveolar
infiltrates, and a presumptive diagnosis of community-acquired pneumonia was made. He was treated
with oral azithromycin without effect, followed by a 10-day course of levofloxacin, also without
effect. During the course of his illness he has lost 4.5 kg without significant anorexia. He is a lifetime
nonsmoker and works as an office manager. He has no pets and no unusual hobbies. On physical
examination, his vital signs are normal, except of respiration rate of 22/min. He is in mild respiratory
distress on exertion. On examination of the chest, bilateral crackles are audible, without wheezing.
Chest radiograph shows bilateral alveolar infiltrates, which are changed in location from those seen
on his original radiographs. Pulmonary function tests show forced expiratory volume in 1 sec (FEV1)
75% of predicted, forced vital capacity (FVC) 72% of predicted, total lung capacity 80% of
predicted, and diffusing lung capacity for carbon monoxide 65% of predicted. Arterial blood gas
values, with the patient breathing room air, are PO2 62 mm Hg, PCO2 42 mm Hg, and pH
7.39.Which of the following is the most likely diagnosis?
Cryptogenic organizing pneumonitis
Hypersensitivity pneumonitis
Resistant pneumococcal pneumonia
Chlamydia pneumonia
Bronchoalveolar cell carcinoma
Patient has severe attack of bronchial asthma which lasts more than 1 hour. Usage of beta-agonists in
inhalation, euphylline intravenously and cholynolytics was not effective. What medicines are
necessary for emergency therapy?
Glucocorticosteroids intravenously
Beta-agonists intravenously
Inhaled glucocorticosteroids
Antihystaminic
Nonsteroid anti-inflammatory medicines
Patient I., a 50 years old man, is evaluated in the emergency department because of fever,
nonproductive cough and 2-day history of myalgia and headache. He has also had nausea and
diarrhea. He is a heavy smoker. On physical examination, he is slightly disoriented. Temperature is
38.9 C, pulse rate is 110/min, respiration rate is 32/min. Chest radiograph shows fluffy infiltrates to
the right upper and lower lobes. Results of laboratory testing show serum sodium of 128 meq/L,
blood urea nitrogen of 42 mg/dL, serum creatinine of 2.2 mg/dL, and serum creatine kinase of 250
U/L. Which one of the following is best next step in the management of this patient’s pneumonia?
Initiate empiric antibiotic therapy for Legionella
Order direct fluorescent antibody testing of the sputum for Legionella
Order serologic testing for Legionella
Send a urine specimen for measurement of Legionella antigen
E.
50.
A. *
B.
C.
D.
E.
51.
A.
B. *
C.
D.
E.
52.
A.
B.
C. *
D.
E.
53.
A.
B.
C.
D.
E. *
54.
A. *
B.
C.
D.
E.
All of the above
Patient J., 36 y.o., complains for fever (39 C), pain in the left part of the chest. Pleuropneumonia was
diagnosed in the patient. What onset is typical for pleuropneumonia?
Acute
Latent
Fulminant
Gradual
Non of the above
Patient K., 27 y.o., complains for dry cough, hoarseness, general weakness, sweating, increase of
body temperature up to 37,5 С. Data of physical examination: vocal fremitus is not changed, resonant
pulmonary sound is heard above the lungs. Preliminary diagnosis?
Acute purulent bronchitis
Acute catarrhal bronchitis
Pneumonia
Pulmonary emphysema
Pleural empyema
Patient K., 37 years old, worker of the factory, during a fire appeared in the area of high
concentration of CO. Into the clinic he was delivered without consciousness. In a blood test: Er - 4,5
x 1012/l, НЬ - 136 g/l, Le - 17,2 x 109/l, Eos - 0 %, band neutrophils - 5 %, segments - 65 %,
lymphocytes - 22 %, monocytes - 3 %, ESR - 3 mm/h, carboxyhaemoglobin in blood - 52 %. What
criterion is the most important for determination the severity degree of the patient’s state?
Results of ECG and spirography
Presence of violations of breathing
Duration of unconsciousness
Prevalence of trophic violations
Development of vascular violations
Patient M., 30 years old, during last 3 years works as a nurse in manipulations cabinet. Last year
during the contact with penicilline she started tp complain on discomfort in throat, sneezing, attack of
cough and dyspnea which disappear after inhalation of salbutamol. During last months attacks of
dyspnea became more severe and occurred only at contact with penicilline. During the life she had
not any diseases including allergic. She hadn’t received antibiotics. Can we consider the bronchial
asthma is professional in this patient?
no, we can’t
yes, we can if we have conclusion about attacks of bronchial asthma
yes, we can if we have conclusion about appearance of bronchial asthma attacks after contact with
penicilline
yes, we can
yes, we can, if allergic and immunological tests are positive
Patient M., 39 years old, complains on attacks of cough with yellow-brown sputum, pain in a right
side, related to the deep breathing, sweating. He is ill for 6 days, after overcooling. Used aspirin.
Objectively: T - 39,6 °С, breathing rate - 26/min, pulse - 110/min, BP -110/70. In lower part of right
lung - moist loud rales. X-ray: in right lower lobe there is massive unhomogeneous infiltration with
lighter areas, sinus is not changed. What complication of disease is the most possible?
Abscesses
Dry pleurisy
Empyema of pleura
Spontaneous pneumothorax
Pulmonary athelectasis
55.
A.
B.
C.
D. *
E.
56.
A. *
B.
C.
D.
E.
57.
A. *
B.
C.
D.
E.
58.
A. *
B.
C.
D.
E.
59.
A. *
B.
C.
D.
E.
60.
A. *
B.
Patient N., 45 y.o., complains for fever (38 C), sweating, dry caugh and general weakness.
Bronchopneumonia was diagnosed in the patient. What onset is typical for bronchopneumonia?
Acute
Latent
Fulminant
Gradual
Non of the above
Patient O., 29-yr-old male prostitute, has felt generally unwell for 2 months with some weight loss.
Over the last 3 weeks he has noticed a dry cough with increasing breathlessness. Two courses of
antibiotics from the GP have produced no improvement. The CXR shows bilateral interstitial
infiltrates. What is the most possible etiology of disease?
Pneumocystis carinii
Streptococcus pneumoniae
Mycoplasma pneumoniae
Fungi
Legionella pneumoniae
Patient of 23, during viral respiratory infection used 1 gram of aspirin, after that he received an attack
of severe dyspnea with prolonged expiration, prescription of euphylline was necessary. There were
no allergic diseases in his history. He had two operations for the treatment of nasal poliposis. What is
your diagnosis?
Aspirin asthma
Symptomatic bronchial spasm
Intermittent bronchial asthma
Persistent bronchial asthma
Asthma of physical exertion
Patient of 44, episodically in spring has dyspnea with worsening of expiration, wheezes in lungs.
Brief daily symptoms are rarer than once a week, night symptomes occur less than 2 times a month.
PEV and FEV1 - 80 %. Between exacerbations wheezes in lungs are absent. What is the possible
diagnosis? bronchial asthma
Intermittent
Easy persistent bronchial asthma
Middle persistent bronchial asthma
Severe persistent bronchial asthma
COPD
Patient of 51, with 10-years history of bronchial asthma, develpos more frequent attacks of dyspnea
and inhalations of astmopent and berotec are not effective. From prescription of what medicine is it
better to begin intensive treatment?
Glucocorticoids
Oxygen therapy
Bronchodylators
Infusion therapy
Heart glycosides
Patient of 54, complaints on dyspnea during insignificant physical exertion, cough with minimal
amount of sputum. Objectively: diffuse cyanosis. Thorax of emphysematous form. In lungs breathing
is vesicular, weakened with prolonged expiration, dry wheezes are heard. BP -140/80. Pulse - 92/min,
rhythmic. Spirography: FVC – 72 %, FEV1/FVC - 50 %. What is the type of respiratory failure in
this patient?
Obstructive
Mixed type with prevalence of obstruction
C.
D.
E.
61.
A. *
B.
C.
D.
E.
62.
A. *
B.
C.
D.
E.
63.
A.
B.
C.
D.
E. *
64.
A. *
B.
C.
D.
E.
65.
A. *
B.
C.
D.
E.
66.
A. *
B.
C.
Restrictive
Mixed type with prevalence of restriction
Respiratory insufficiency is absent
Patient P., a young male homosexual with Kaposi's sarcoma, complains of increasing breathlessness
and a dry cough. He has a 3-day history of shivering, general malaise and productive cough. The
X-ray shows right lower lobe consolidation. What is the most possible etiology of disease?
Pneumocystis carinii
Mycoplasma tuberculosis
Haemophilus influenzae
Chlamydia trachomatis
Klebsiella pneumoniae
Patient Q., 37 years old, was operated in the surgical department because of appendicitis. After 4
days appeared the recidive of chills, cough, dyspnea, fever 38,5 °С, returned leucocytosis with shift
to the left in leucocyte formula. On X-ray there is infiltration of lower right lobe. What is the
diagnosis?
Nosocomial pneumonia
Pulmonary abscess
Infarction pneumonia
Community-acquired pneumonia
Tuberculosis
Patient R., 48 y.o., complains for sharp pain in the right part of his chest at deep breathing and cough.
Pain in the chest which relates to disease of respiratory system, is typical for
Bronchiectatic disease
Asthma
Emphysema
Exudative pleurisy
Dry pleurisy
Patient S., a 25-year-old male has just returned from holiday abroad, presents with flu-like illness,
headaches, high fever. Prior to this, he had complained of abdominal pain, vomiting, diarrhea
associated with blood per rectum. What is the most possible etiology of disease?
Legionella pneumoniae
Streptococcus pneumoniae
Mycoplasma pneumoniae
Pneumocystis carinii
Fungi
Patient V., a 24 years old barman, presents with a dry cough of sudden onset. He complains of a chest
pain and rusty sputum. He also has a very high fever, rapid breathing, cyanosis and crepitations.
Pneumonia was suspected. What is the most nesessary method of investigation?
Chest X-ray
Spirography
Analysis of sputum
General blood analysis
General urine analysis
Patient W., 62 y.o., suffers with morning cough with expectoration of large volume of greenish
sputum. Sputum is better expelled in a certain position of patient’s body. Such kind of cough is
typical for
bronchiectatic
disease
asthma
emphysema
D.
E.
67.
A. *
B.
C.
D.
E.
68.
A. *
B.
C.
D.
E.
69.
A. *
B.
C.
D.
E.
70.
A. *
B.
C.
D.
E.
pneumonia
pleurisy
Patient W., 67 years old, during the epidemic of influenza after decreasing of fever noticed pain that
appeared in a thorax, cough with yellow-green sputum (amount-100 ml a day), sometimes with some
blood. Objectively: breathing rate - 36/min. In lungs from the right side lower scapula there is dull
sound during percussion, hard breathing, and moist rales. Blood test: L - 18,6х109/l, ESR -64 mm/h.
Analysis of sputum: L -80-100 , Er - 40-50, elastic fibres, cocci. X-ray: rhadicis are enlarged, from
the right side lower lobe is heterogeneously infiltrated with two lighter areas. What is the most
possible previous diagnosis?
Right-side pneumonia with abscesses
Peripheral cancer
Infiltrative tuberculosis in the phase of disintegration
Exudative pleurisy
Infarction-pneumonia
Patient X., 55 years old, was admitted to the hospital recently. He complaints on cough with very
small amount of mucous-purulent sputum, significant weakness, increased temperature, which is
accompanied with chill, dizziness. Objectively: t - 38°С. Breathing rate - 22/min. Heart rate - 90/min,
BP - 110/70. From the right side below scapula the vocal fremitus is increased, percussion sound is
shortened, vesicular breathing is weakened, small amount of moist rales. Tones of heart are dull,
rhythm is correct, moderate tachycardia. Doctor suspected pneumonia. The presence of what
syndrome let to suspect such diagnosis?
Pulmonary tissue infiltration
Intoxication
Inflammation
Bronchial obstruction
Respiratory insufficiency
Patient Z., a 33-yr-old car mechanic, is brought to casualty by his girlfriend. She describes a 2-day
history of rigors, sweats and intermittent confusion. On examination he is agitated, sweaty and
pyrexial with 38.6° C. He is hyperventilating and cyanosed despite receiving O2 by face mask. There
is dullness to percussion and bronchial breathing at the left lung base. What method of investigation
is necessary?
Chest
X-ray
Spiral CT with contrast
Arterial blood gases
Blood count and film
Urea and electrolytes of blood
Previously healthy 28-year-old man is evaluated in the emergency department because of fever,
productive cough, and shortness of breath. His temperature is 40 C, pulse is 120/min, respiration rate
is 32/min, and blood pressure is 100/70 mm Hg. Measurement of arterial blood gases with the patient
breathing room air shows PO2 of 55 mm Hg, PCO2 of 30 mm Hg, pH of 7.41. Chest radiograph
reveals bilateral alveolar infiltrates with no effusions. Gram stain of the sputum reveals gram-positive
diplococci. Which of the following is the most appropriate for this patient?
Hospitalize him
Treat him as an outpatient with oral therapy
Treat him as an outpatient with intravenous therapy
Hospitalize the patient in the intensive care unit
All of the above
71.
A. *
B.
C.
D.
E.
72.
A.
B.
C.
D. *
E.
73.
A.
B. *
C.
D.
E.
74.
A.
B. *
C.
D.
E.
75.
A.
B. *
C.
D.
E.
76.
A.
B.
Previously healthy 32-yr-old woman presents with general malaise, severe cough and breathlessness,
which has not improved with a 7 day-long course of Amoxycillin. There is nothing significant to find
on examination. The X-ray shows patchy shadowing throughout the lung fields. The blood film
shows clumping of red cells with suggestion of cold agglutinins. What is the most possible etiology
of disease? pneumoniae
Mycoplasma
Legionella pneumonia
Haemophilus influenzae
Chlamydia trachomatis
Klebsiella pneumoniae
The 60-year-old patient tells you that she smoked three packs of cigarettes per day since she was 15
years old until she was 40, and then smoked two packs per day. How many pack-years should you
record in the patient's history?
45
80
90
115
Non of above
The 82-year-old patient has a pulmonary infection. Which action addresses the age-related change of
increased vascular resistance to blood flow through pulmonary vasculature in this patient?
Encouraging the patient to turn, cough, and deep breathe every hour
Assessing the patient's level of consciousness
Raising the head of the bed
Humidifying the oxygen
Non of above
The patient has broken ribs that penetrated through the skin as a result of a motor vehicle crash 3
days ago. The patient now complains of increased pain, shortness of breath, and fever. Which
assessment finding alerts the physician to the possibility of a pleural effusion and empyema?
Wheezing on exhalation on the side with the broken ribs
Absence of fremitus at and below the site of injury
Crepitus of the skin around the site of injury
Absence of gastric motility
Non of above
The patient has severe nasal congestion, headache, and sneezing but no rhinorrhea, watery eyes, sore
throat, or fever. Which statement made by the patient alerts the physician to the possibility of rhinitis
medicamentosa?
“I have been taking two aspirins every 6 hours for this headache.”
“My nose doesn't stay open even though I'm using nasal spray every hour.”
“I have been taking a lot of vitamin C this year to keep from getting so many colds.”
“The only way I can get to sleep with this nasal congestion is by taking an over-the-counter
antihistamine at night.”
Non of above
The patient is 34 years old and has been diagnosed with COPD as a result of being homozygous for a
mutation of the alpha1-antitrypsin (AAT) gene alleles. His wife has two normal AAT gene alleles.
He is concerned that his two children may develop this problem. What is your best response?
“Because neither of your parents have COPD and your wife does not have the abnormal gene alleles,
your children will not be affected.”
“Because your wife is not affected nor is or a carrier, your children will have normal levels of AAT
and their risk is the same as for the general population.”
C.
D. *
E.
77.
A.
B.
C.
D. *
E.
78.
A. *
B.
C.
D.
E.
79.
A. *
B.
C.
D.
E.
80.
A. *
B.
C.
D.
E.
81.
A. *
B.
C.
“Because you have the mutations and your wife does not, your son will be at an increased risk for
developing COPD but your daughter will only be a carrier.”
“Because both of your AAT gene alleles are mutated, your children will each have one abnormal
gene and their risk for COPD is only increased if they smoke or are chronically exposed to other
precipitating factors.”
Non of above
The patient is a 42-year-old man recently diagnosed with new-onset asthma. What specific
patiental/demographic information should you obtain related to this diagnosis?
Previous diagnosis of pneumonia or tuberculosis
Known allergies and hypersensitivities
Nutritional intake and diet history
Occupation and hobbies
Non of above
The patient with hospital-acquired (nosocomial) pneumonia caused by a bacterial infection with a
gram-negative microorganism is receiving treatment with intravenous amikacin (Amikin). In addition
to frequent respiratory assessment, what other assessment should the physician routinely perform to
identify a common complication of this medication?
Monitor urine output every shift
Perform neuro checks every 2 hours
Examine the stool and vomitus for the presence of blood
Monitor the complete white blood cell count and differential daily
Non of above
Woman 36 y.o., complaints on dry cough, dyspnea. Felt ill after viral respiratory infection 2 years
ago. Objectively: breathing rate - 16/min, pulse -68/min, BP - 130/90. In lungs during percussion clear pulmonary sound. Auscultation – diffuse dry wheezes. To check reversibility of bronchial
obstruction it is necessary to provide test with:
Salbutamol
Forced expiration
Obzidan
Physical exertion
Oxygen
Woman 45 y.o., is ill with bronchial asthma for 20 years. She came to pulmonologist to discuss plan
of treatment in different cases. Now attacks arise 2-3 times a week, she uses intal, ventolin if
necessary. What medicine would be useful in case of arising night attacks?
Inhaled glucocorticosteroids
To use intal
To continue usual therapy
To use prednisole
To add atrovent to usual treatment
Woman 58 y.o., is ill with bronchial asthma, entered the hospital with complaints on dyspnea and
palpitation. Objectively: condition is severe, breathing is noisy with participation of additional
breathing muscles, periodically cramps are present, diffuse cyanosis. In lungs – diffuse dry wheezes,
breathing is weaker in the lower parts of lungs. Pulse - 100/min, liver is a little lower from the edge
of costal arch, edema, 3 extrasystoles/min, BP - 140/100, РаO2 - 45, pH - 7,3. What syndrome is the
most severe for this patient?
Respiratory failure
Blood hypertension
Tachycardia
D.
E.
82.
A. *
B.
C.
D.
E.
83.
A. *
B.
C.
D.
E.
84.
A. *
B.
C.
D.
E.
85.
A. *
B.
C.
D.
E.
Arrhythmia
Heart failure
Woman of 34, for 15 years is ill with bronchial asthma. Some time ago increased frequency of
attacks of dyspnea, they arised 4-5 times a week, night attacks - 2-3 times a month. Used salbutamol
to remove attacks. Objectively: condition is satisfactory. Breathing rate - 20/min. Heart rate - 76/min,
BP - 120/80. In lungs there is vesicular breathing. Tones of heart are a little weak, rhythm is normal.
What medicine must be used for the prophylaxis of attacks of bronchial asthma on the first stage?
Corticosteroids in inhalations
Corticosteroids in injections
Regular usage of salbutamol
Corticosteroids orally
Cromoglicat sodium
Woman of 62, is ill with bronchial asthma. Recently appeared pain behind the sternum, interruptions
in work of heart. Objectively: t - 36,6 °С, pulse -78/min, extrasystoles are present, BP -160/95,
breathing rate - 18/min. In lungs during auscultation - breathing with prolonged expiration, diffuse
dry wheezes are present. What preparations are not indicated in this situation?
?-blockers
Ca-channel blockers
Nitrosorbid
Sustak
Anticoagulants
Woman, 68 years old, with moderate emphysema is evaluated during a routine visit. She has chronic
dyspnea on exertion but has no cough or sputum production. She uses supplemental oxygen, 2 l/min,
when sleeping and on exertion. She currently uses albuterol and ipratropium four times per day, and
salmeterol and theophylline twice per day. She is currently enrolled in a pulmonary rehabilitation
program and is concerned about “catching a cold” from other people enrolled in the pulmonary
rehabilitation program. What is the best advice for this patient?
Practice good hand washing, attempt to avoid close prolonged contact with ill persons, and take
pneumococcal and annual influenza vaccine
Avoid any social functions where there will be large crowds
Discontinue attendance at the pulmonary rehabilitation program
Take a daily antibiotic (long-term suppressive antibiotic therapy) to prevent pneumonia
All of the above
Woman, a 76 years old resident of a nursing home, is evaluated in the emergency department because
of decreasing mental status and hypothermia. She has a history of stroke and is currently taking only
aspirin. She has been able to eat on her own and there have been no witnessed aspirations. She has
not been treated recently with antibiotics. Her leukocyte count is 12,000/l, and her hemoglobin is 120
g/l. Serum electrolytes are within normal limits and she has mild chronic renal insufficiency. Chest
radiograph shows a small interstitial infiltrate in the right lower lung field. She receives traditional
empiric treatment for community-acquired pneumonia. Therapy for which of the following should
also be considered?
Enteric gram-negative organisms
Pseudomonas aeruginosa
Anaerobic bacteria
Aspergillus fumigatus
Mycobacterium tuberculosis
86.
A. *
B.
C.
D.
E.
87.
A. *
B.
C.
D.
E.
88.
A. *
B.
C.
D.
E.
89.
A. *
B.
C.
D.
E.
90.
A. *
B.
C.
D.
67-year-old man with chronic obstructive pulmonary disease is evaluated because of chronic
dyspnea, minimally productive cough, and limited exercise tolerance. He thinks his dyspnea on
exertion has worsened. He stopped smoking cigarettes 8 years ago and is currently using an
ipratropium inhaler four times per day and salmeterol discus twice per day. His body mass index,
which 6 months ago was 21, is now 19. On physical examination, he is afebrile, his pulse rate is
94/min and regular, and respiration rate is 20/min. His breathing is unlabored at rest. He has signs of
chest hyperinflation and decreased breath sounds without wheezing. He has no peripheral edema. The
remainder of his examination is normal; results of a fecal occult blood test are negative. Baseline
spirometry is unchanged. Forced expiratory volume in 1 sec (FEV1) 35% of predicted Forced vital
capacity (FVC) 85% of predicted FEV1/FVC ratio50% PO2 62 mm Hg PCO2 45 mm Hg pH 7.38
(with the patient breathing room air) Chest radiograph reveals only hyperinflation. What is the best
Refer
for pulmonary
rehabilitation
with exercise and nutritional counseling.
way tohim
manage
this patient’s
weight loss?
Provide dietary instructions to increase his caloric intake.
Treat him with anabolic steroids
Prescribe oxygen supplementation to improve his oxygen consumption.
Add inhaled corticosteroids to his medical regimen.
68-year-old man with severe chronic obstructive pulmonary disease (forced expiratory volume in 1
sec 32% of predicted) is evaluated because of severe dyspnea and the inability to carry out his
activities of daily living. He is on maximal bronchodilator and oxygen therapy. Which of the
following might pulmonary rehabilitation improve?
Exercise tolerance
Forced expiratory volume in 1 sec
Oxygenation
Survival
Non of above
A 35-yr-old accountant presents with a chronic cough, dyspnea and wheezing. He produces copious
sputum. His arterial carbon dioxide is low and his arterial oxygen is normal. Chest X-ray: high
pneumatization of lungs. What is your diagnosis?
Emphysema
Bronchogenic carcinoma
Pulmonary embolism
Pneumonia
Tuberculosis
A 53-yr-old smoker with chronic cough and copious yellow sputum presents in a state of agitation.
He is confused. His pulse is bounding. He has a terrible headache and you find papilloedema on
fundoscopy. What is the cause?
Emphysema
Bronchogenic carcinoma
Pneumonia
Cardiac failure
Bronchial asthma
A 53-yr-old smoker with chronic cough and copious yellow sputum presents in a state of agitation.
He is confused. His pulse is bounding. He has a terrible headache and you find papilloedema on
fundoscopy. What may be found on chest X-ray?
Signs of fibrosis and hyperpneumatization
Infiltration
Round shadow
Round shadow with horizontal level of fluid
E.
91.
A. *
B.
C.
D.
E.
92.
A. *
B.
C.
D.
E.
93.
A. *
B.
C.
D.
E.
94.
A. *
B.
C.
D.
E.
Signs of fibrosis and local infiltration
A 55-year-old man is evaluated in the emergency department because of a 5-day history of increased
dyspnea and cough productive of yellow-green mucus. Nine months ago, he required prolonged
mechanical ventilation for an exacerbation of chronic obstructive pulmonary disease. His medical
history includes hypertension. On recent pulmonary function testing, the forced expiratory volume in
1 sec (FEV1) was 38% of predicted. His temperature is 38.1 C, pulse rate is 135/min and irregular,
respiration rate is 25/min, and blood pressure is 90/65 mm Hg. He is mildly lethargic but arousable
and oriented. He has a weak cough with pooling of secretions in the oral cavity and hypopharynx.
Electrocardiogram demonstrates multifocal atrial tachycardia. Chest examination reveals accessory
muscle use, coarse rhonchi, and decreased breath sounds at the right base. Leukocyte count is 1
7,000/?L. Chest radiograph shows right lower lobe consolidation. With the patient breathing 4 L
oxygen, arterial blood gases show a PO2 of 50 mm Hg, a PCO2 of 65 mm Hg, and a pH of 7.25.
Therapy with methylprednisolone sodium succinate, 125mg intravenously every 6 h, is initiated,
along with nebulized albuterol and ipratropium bromide every 4 h, and azithromycin, 500 mg
administered intravenously daily. Which of the following is the most appropriate additional
Intubate
and begin mechanical ventilation
management?
Initiate mucloytic therapy, chest physiotherapy, and oral-tracheal suctioning
Administer a helium-oxygen mixture of 70%:30%, delivered by face mask
Initiate bilevel noninvasive positive-pressure ventilation by face mask
Administer 40% oxygen by Venturi mask
A 55-year-old man is evaluated in the emergency department because of an acute, severe asthma
attack; he is hospitalized in the intensive care unit for aggressive medical therapy and monitoring. He
is expectorating thick greenish sputum. His medical history includes hypertension, cholecystectomy,
and glaucoma. Chest radiograph reveals hyperinflation only. Medical therapy in the emergency
department included repeated doses of aerosolized albuterol and ipratropium, as well as
methylprednisolone, 125mg administered intravenously. Peak expiratory flow rate is unimproved at
80 L/min. Which of the following is the most appropriate next step in this patient’s management?
Intravenous magnesium sulfate
Nebulized ipratropium bromide administered by face mask
Broad-spectrum antibiotics targeting community-acquired respiratory pathogens
Inhaled corticosteroids
Non of above
A 56-yr-old man wheezes and coughs. He has tried to give up smoking, but he finds it very difficult.
He is thin and healthy looking with a rounded chest. His breathing is noisy. His cough is
unproductive. What method of investigation is not useful?
Ultrasound examination
Chest X-ray
Spirography
General blood analysis
Sputum analysis
A 56-yr-old man wheezes and coughs. He has tried to give up smoking, but he finds it very difficult.
He is thin and healthy looking with a rounded chest. His breathing is noisy. His cough is
unproductive. What treatment has to be prescribed?
Salbutamol
Amoxycillin
Prednisolone
ACC
Bronchial lavage
95.
A. *
B.
C.
D.
E.
96.
A. *
B.
C.
D.
E.
97.
A. *
B.
C.
D.
E.
98.
A. *
B.
A 56-yr-old man wheezes and coughs. He has tried to give up smoking, but he finds it very difficult.
He is thin and healthy looking with a rounded chest. His breathing is noisy. His cough is
unproductive. What is the previous diagnosis?
Emphysema
Bronchogenic carcinoma
Pneumonia
Asthma
Bronchitis
A 57-year-old man with advanced chronic obstructive pulmonary disease (COPD) and systemic
hypertension is evaluated because of a 6-day history of productive cough and shortness of breath. He
uses inhaled albuterol and ipratropium bromide, a long-acting theophylline preparation, and
lisinopril. He uses supplemental oxygen at night and during ambulation. Ciprofloxacin is prescribed
for an exacerbation of COPD. Three days later, having had nausea for a day, the man is brought to
the emergency department after he is found nearly unconscious. Arterial oxygen saturation is 89%,
with the patient breathing room air. Electrocardiogram shows normal sinus rhythm with nonspecific
ST-T changes in the lateral chest leads. Which of the following is likely to have interacted with
ciprofloxacin and caused the symptoms that brought the man to the emergency department?
Theophylline
Albuterol
Ipratropium bromide
Lisinopril
Oxygen
A 59-year-old man with advanced chronic obstructive pulmonary disease is evaluated because of a
daily cough productive of white or yellow sputum, dyspnea after climbing one flight of stairs, and a
recent 4.5-kg weight loss with no associated change in appetite or food intake. The patient stopped
smoking 4 years ago. On physical examination, he has diminished breath sounds throughout all lung
fields. Arterial oxygen saturation measured by pulse oximetry with the patient at rest, breathing room
air, is 87%. Chest radiograph suggests hyperinflation of the lungs but shows no pulmonary infiltrates
or abnormalities of the cardiac silhouette. Pulmonary function studies show a forced expiratory
volume in 1 sec 39% of predicted and forced vital capacity 78% of predicted. Which of the following
may prolong life in this patient?
Supplemental oxygen
Albuterol
Ipratropium bromide
Theophylline
Lisinopril
A 60-year-old woman is hospitalized for an exacerbation of chronic obstructive pulmonary disease.
She is treated with ipratropium bromide by nebulizer every 4 hours; intravenous azithromycin, 500
mg/d; methylprednisolone, 125 mg intravenously every 6 hours; and oxygen by nasal cannula.
During the first 2 hospital days, her condition remains unchanged. On the 3rd hospital day, she
develops increased dyspnea and a cough productive of sputum. On physical examination, she is
awake and alert and in moderate respiratory distress. Her temperature is 36.7 C, pulse rate is
110/min, respiration rate is 20/min, and blood pressure is 150/90 mm Hg. Her lungs are
hyperresonant to percussion, with accessory muscle use, poor air movement, mild wheezing, and no
crackles. Chest radiograph demonstrates hyperinflation, with no other abnormalities. Leukocyte
count is 16,000/L. Arterial blood gas measurements, with the patient breathing 1.5 L oxygen, show
PO2 of 55 mm Hg, PaCO2 of 55 mm Hg, and pH of 7.32. She is transferred to the intensive care unit
for close observation and possible assisted ventilation. Which of the following is most appropriate
Add
albuterol
to ipratropium
bromide
by nebulizer every 4 hours
additional
management
for this
patient?
Discontinue azithromycin and begin levofloxacin
C.
D.
E.
99.
A. *
B.
C.
D.
E.
100.
A. *
B.
C.
D.
E.
101.
A. *
B.
C.
D.
E.
Begin intravenous aminophylline
Increase methylprednisolone to 250 mg every 6 hours
Increase oxygen by nasal cannula to 3 L/min
A 66-year-old woman with chronic obstructive pulmonary disease is evaluated because of chronic
cough and dyspnea. She currently uses a long-acting bronchodilator twice per day, an inhaled
corticosteroid twice per day, ipratropium four times per day, and albuterol four to six times per day.
She smokes 1 pack of cigarettes per day. On physical examination, her vitals signs are normal. Her
oxygen saturation at rest and with exertion is 94%. She has diminished breath sounds, a prolonged
expiratory-to-inspiratory phase, and no wheezes. Her heart rate and rhythm are regular, with
physiologically split S2, and no murmurs or rubs. Chest radiograph reveals hyperinflation, increased
retrosternal airspace, and flattened hemidiaphragms bilaterally. Which of the following should be
initiated at this time to address this patient’s cough and dyspnea?
Discuss techniques to help her to quit smoking
Increase her use of the long-acting bronchodilator.
Prescribe supplemental oxygen
Provide emergency treatment for a tension pneumothorax
Increase her dosage of inhaled corticosteroid.
A 67-year-old man is evaluated because of a 3-week history of cough productive of blood-streaked
sputum. A chest radiograph shows an infiltrate in the right upper lobe. He is treated with antibiotics
for 2 weeks, but the blood-streaked sputum persists. A CT scan of the chest shows a mass obstructing
the right upper lobe and evidence of postobstructive pneumonitis. Examination of the mediastinum
shows enlarged lymph nodes in the right paratracheal space. A positron emission tomography (PET)
scan shows uptake in the mass itself and in the lymph nodes in the right paratracheal space.
Fiberoptic bronchoscopy is performed; an endobronchial lesion is identified and a sample is taken for
biopsy. The biopsy shows squamous cell carcinoma. Which of the following is the best next step in
this patients management?
Perform immediate mediastinoscopy
Refer him for radiation therapy
Refer him for surgery
Perform a repeat positron emission tomography scan
Non of above
A 70-year-old man is evaluated because of shortness of breath. He has noted progression of his
symptoms, primarily with exertion over the past 6 months. He is unable to walk one flight of stairs or
two blocks on level ground without becoming short of breath. He has no chest pain, paroxysmal
nocturnal dyspnea, orthopnea, or lower extremity edema. He has a 40-pack-year history of cigarette
smoking, but stopped smoking 10 years ago. He worked in a naval shipyard 50 years ago but has
spent most of his working life as a schoolteacher. On physical examination, his respiration rate is
18/min but he does not appear short of breath. Examination of the chest shows an increased
anterior-posterior diameter. On auscultation he has diffusely decreased breath sounds and a
prolonged expiratory phase with no wheezing. He has no cyanosis or clubbing. Spirometry shows
forced expiratory volume in 1 sec 55% of predicted and forced vital capacity 80% of predicted with a
ratio of FEV1 to FVC 60%. Which of the following is the best test to evaluate this patients condition?
Lung volumes and diffusing capacity
Echocardiography
Exercise spirometry
Methacholine challenging testing
Non of above
102.
A. *
B.
C.
D.
E.
103.
A. *
B.
C.
D.
E.
104.
A. *
B.
C.
D.
E.
105.
A. *
B.
C.
D.
E.
106.
A. *
B.
C.
D.
A 71-year-old woman is evaluated because of progressive fatigue, weakness, and dyspnea on
exertion. A former smoker, she has a history of advanced emphysema and is on continuous long-term
oxygen therapy. She currently uses a long-acting bronchodilator twice per day, theophylline and an
inhaled corticosteroid twice per day, ipratropium four times per day, and albuterol four to six times
per day. Physical examination is normal. Heart and lung examinations are consistent with
long-standing, advanced emphysema. Her laboratory and radiographic findings are unremarkable.
Which of the following is the best next step in this patient’s management?
Refer her to a multidisciplinary rehabilitation program
Prescribe an antidepressant medication
Prescribe an empiric course of “pulse dose” corticosteroids at 500 mg/d for 3 consecutive days
Refer her for surgery
Non of above
A middle-aged smoker presents with chronic cough and phlegm. His sputum is tenacious but not
yellow or blood stained. His chest is hyperinflated. His arterial carbon dioxide is high and is arterial
oxygen is low. What is the previous diagnosis?
Emphysema
Bronchitis
Bronchogenic carcinoma
Pneumonia
Respiratory failure
For 8 years patient is disturbed with cough in the morning with little amount of sputum, shortness of
breath. He is a smoker for more than 10 years. Objective examination: cyanosis, increased duration
of expiration, dry wheezes. Possible diagnosis is:
COPD
Pneumonia
Idiopathic alveolitis
Bronchiectasis
Bronchial asthma
Man 39 y.o., driver, complaints on the shortness of breath during physical exertion, cough with the
small amount of light sputum mostly in the morning. For a long time is ill with COPD. He is a
smoker, uses alcohol episodically. Objectively: temperature - 36,5 °С, breathing rate - 24/min, pulse 90/min, BP - 120/ 80. During auscultation breathing is hard, moderate amount of dry wheezes. FEV1
- 68 % of normal index. What methods are necessary to prevent the disease?
To stop smoking
To change the job
To avoid alcohol
Sanation of chronic infection
To change region of living
Man 60 y.o., complains on expiration dyspnea which increases at the physical exertion, cough with
small amount of mucus-purulent sputum mostly in the morning. He is ill with COPD. Objectively:
temperature - 36,0 °С, breathing rate - 22/min, pulse - 84/min, BP - 110/70. Skin is moist, diffuse
cyanosis. Auscultation: breathing is hard, diffuse dry and moist wheezes are present. FEV - 62 %;
pharmacological test with atrovent showed 5 % increasing of this index. What mechanism of
bronchial obstruction development is the most possible in this case?
Diffuse sclerotic changes
Hypercrynia
Inflammatory edema
Bronchial spasm
E.
107.
A. *
B.
C.
D.
E.
108.
A. *
B.
C.
D.
E.
109.
A. *
B.
C.
D.
E.
110.
A. *
B.
C.
D.
E.
111.
A. *
B.
C.
D.
E.
112.
A. *
Mucostasis
Man of 43 complaints on dyspnea during physical activity. Objectively: temperature 36,4 °С,
breathing rate - 20/min, pulse - 78/min, BP-125/80. Emphysematous form of thorax. In lungs – weak
vesicular breathing. What research must be provided by patient at home to decide question about
efficiency of prescribed broncholytics?
Peakflowmetry
Spirography
ECG-control of overload of right departments of heart
Bronchoscopy
Analysis of sputum
Patient 44 y.o., complaints on attack of dyspnea, which arises suddenly at night. Connects this attack
with overcooling. He is ill for more than 10 years. Thorax of emphysematous form. During
percussion in lungs – “box” sound. During auscultation there is plenty of dry wheezes. In blood:
moderate leucocytosis, eosinophylia - 10 %. On the X-ray film – increased pneumatization of
pulmonary tissue. What diagnosis is the most possible one?
Bronchial asthma, exacerbation phase
Bronchiectasis, exacerbation phase
COPD, exacerbation phase
Chronic bronchitis
Eosinophylic pulmonary vasculitis
Patient 47 y.o., complaints on cough, dyspnea during physical exertion, local pain in the heart region,
general weakness. Suffers with COPD for 10 years. During auscultation of lungs were founded
disseminated dry wheezes. Systolic blood pressure in the pulmonary artery is 50. It is the most
important to prescribe for treatment:
Euphyllin
Bromhexin
Caffeine
Prednisone
Atropin
Patient complaints on attacks of dyspnea, which arises 1-2 times a week, night symptoms - 2 times a
month and even more frequent. For a patient night sleep is broken as a result of attacks of dyspnea.
FEV1 > 80 % from normal. What diagnosis would you suspect?
Mild persistent BA
Severe persistent BA
Intermittent BA
Moderate persistent BA
Status asthmaticus
A 14-yr-old student with cystic fibrosis rapidly deteriorated and developed acute respiratory failure
while in hospital. Which infection is the most possible cause of deterioration of his state?
Pseudomonas aeroginosa
Pneumocystis carinii
Chlamydia psittaci
Ecoli
Mycobacterium tuberculosis
A 20-yr-old male IV drug abuser presents with breathlessness and cough. CXR reveals patchy areas
of consolidation with abscess formation. Which infection is the most possible cause of his state?
Staphylococcus aureus
B.
C.
D.
E.
113.
A. *
B.
C.
D.
E.
114.
A. *
B.
C.
D.
E.
115.
A. *
B.
C.
D.
E.
116.
A. *
B.
C.
D.
E.
117.
A. *
B.
C.
Chlamydia psittacci
Coxiella burnetti
Pneumocystis carinii
Pseudomonas aeroginosa
A 20-yr-old previously healthy woman presents with general malaise, severe cough and
breathlessness, which has not improved with a 7 day course of Amoxycillin. There is nothing
significant to find on examination. The X-ray shows patchy shadowing throughout the lung fields.
The blood film shows clumping of red cells with suggestion of cold agglutinins. Which infection is
the most possible cause of her state?
Mycoplasma pneumoniae
Legionella pneumonia
Haemophilus influenzae
Chlamydia trachomatis
Klebsiella pneumoniae
A 22-year-old previously healthy man is evaluated in the emergency department because of fever,
productive cough, and shortness of breath. His temperature is 40 C, pulse is 120/min, respiration rate
is 32/min, and blood pressure is 100/70 mm Hg. Measurement of arterial blood gases with the patient
breathing room air shows PO2 of 55 mm Hg, PCO2 of 30 mm Hg, pH of 7.41. Chest radiograph
reveals bilateral alveolar infiltrates with no effusions. Gram stain of the sputum reveals gram-positive
diplococci. Which of the following is the most appropriate for this patient?
Hospitalize him
Treat him as an outpatient with oral therapy
Treat him as an outpatient with intravenous therapy
Hospitalize the patient in the intensive care unit
All of the above
A 22-yr-old barman presents with a dry cough of sudden onset. He complains of a chest pain and
rusty sputum. He also has a very high fever, rapid breathing, cyanosis and crepitations. Pneumonia
was suspected. What is the most necessary method of investigation?
Chest X-ray
Spirography
Analysis of sputum
General blood analysis
General urine analysis
A 22-yr-old barman presents with a dry cough of sudden onset. He complains of a chest pain and
rusty sputum. He also has a very high fever, rapid breathing, cyanosis and crepitations. What is your
previous diagnosis?
Pneumonia
Asthma
Lung abscess
COPD
Lung cancer
A 24-yr-old car mechanic is brought to casualty by his girlfriend. She describes a 2-day history of
rigors, sweats and intermittent confusion. On examination he is agitated, sweaty and pyrexial with
38.6° C. He is hyperventilating and cyanosed despite receiving O2 by face mask. There is dullness to
percussion and bronchial breathing at the left lung basse. What method of investigation is nesessary?
Chest X-ray
Spiral CT with contrast
Arterial blood gases
D.
E.
118.
A. *
B.
C.
D.
E.
119.
A. *
B.
C.
D.
E.
120.
A. *
B.
C.
D.
E.
121.
A. *
B.
C.
D.
E.
122.
A. *
B.
C.
D.
E.
123.
Blood count and film
Urea and electrolytes
A 24-yr-old man presents dry cough, skin manifestations and bone and muscle aches. His chest
radiograph shows widespread patchy shadows. Blood tests show evidence of haemolysis. Which
infection is the most possible cause of his state?
Mycoplasma pneumoniae
Pneumocystis carinii
Chlamydia psittaci
coli
Pseudomonas aeroginosa
A 27-yr-old male patient has just returned from holiday abroad presents with flu-like illness,
headaches, high fever prior to this, he had complained of abdominal pain, vomiting, diarrhoea
associated with blood per rectum. Which infection is the most possible cause of his state?
Legionella pneumoniae
Streptococcus pneumoniae
Mycoplasma pneumoniae
Pneumocystis carinii
Fungi
A 27-yr-old male prostitute has felt generally unwell for 2 months with some weight loss. Over the
last 3 weeks he has noticed a dry cough with increasing breathlessness. Two courses of antibiotics
from the GP have produced no improvement. The CXR shows bilateral interstitial infiltrates. Which
infection is the most possible cause of his state?
Pneumocystis carinii
Streptococcus pneumoniae
Mycoplasma pneumoniae
Fungi
Legionella pneumoniae
A 28 years old patient, complaints on cough with small amount of colourless sputum, pain in the
right half of thorax during breathing, shortness of breath, increase of temperature to 39 °С. Felt ill
rapidly. Used aspirin. Objectively: herpes on lips. In lower lobe of right lung there is dull percussion
sound, bronchial breathing. Chest X-ray: there is homogeneous infiltration of right lower lobe. What
is the most possible etiology of pneumonia?
Pneumococcus
Staphylococcus
Mycoplasma
Legionella
Klebsiella
A 30-yr-old man with AIDS presents with fever, dry cough and dyspnoea. CXR shows diffuse
bilateral alveolar and interstitial shadowing beginning in the perihilar regions and spreading outward.
Which infection is the most possible cause of his state?
Pneumocystis carinii
Chlamydia psittacci
Coxiella burnetti
Staphylococcus aureus
Pseudomonas aeroginosa
A 35-yr-old previously healthy man returned from holiday five days ago. He smokes 10 cigarettes
per day. He presents with mild confusion, a dry cough and mild pyrexia. His chest is normal. The
X-ray shows widespread upper zone shadowing. Which infection is the most possible cause of his
state?
A. *
B.
C.
D.
E.
124.
A. *
B.
C.
D.
E.
125.
A. *
B.
C.
D.
E.
126.
A. *
B.
C.
D.
E.
127.
A. *
B.
C.
D.
E.
128.
A. *
B.
C.
D.
Legionella pneumoniae
Haemophilus influenzae
Chlamydia trachomatis
Pneumocystis carinii
Klebsiella pneumoniae
A 38 years old patient, who drunk a lot of alcohol, has severe pneumonia. His condition was
worsened, the temperature of body rose to 39-40 °С, an unpleasant smell appeared from a mouth,
increased amount of purulent sputum; increased ESR and amount of band leucocytes. On the X-ray in the lower lobe of right lung there is massive infiltration with bright area in a center. What
complication is it necessary to suspect?
Acute pulmonary abscess
Bronchiectasis
Infarction-pneumonia
Gangrene of lungs
Empyema of pleura
A 40-yr-old man who works in an abattoir presents with sudden onset of fever, myalgia, headache,
dry cough and chest pain. CXR shows patchy consolidation of the right lower lobe giving a ground
glass appearance. Which infection is the most possible cause of his state?
Coxiella burnetti
Chlamydia psittacci
Staphylococcus aureus
Pneumocystis carinii
Aspergillosis
A 44-yr-old travelling insurance salesman presents with high fever myalgia abdominal pain and
haemoptysis. CXR shows diffuse patchy lobar shadows. The cough progresses from a modest
nonproductive cough to producing mucopurulent sputum. The fever persists for 2 weeks. Which
infection is the most possible cause of his state?
Legionella pneumophilia
Mycoplasma pneumoniae
Actinomycosis
Tuberculosis
Streptococcus pneumoniae
A 48 years old patient, complaints on weakness, dyspnea, pain in the left half of thorax, permanent
cough with viscid sputum, in which particles of blood are sometimes determined. For the last 3
months lost 5 kg of body mass. On the X-ray of lungs there is total homogeneous shade is determined
from the left side. Organs of mediastinum are displaced to the left. What diagnosis is possible?
Lung athelectasis
Lungs gangrene
Total exudative pleurisy
Pneumonia
Empyema of pleura
A 48-yr-old man presents with fever, rigors, headache and diarrhea. He recently had been on a
holiday abroad. CXR shows consolidation. Which infection is the most possible cause of his state?
Legionella pneumoniae
Staphylococcus aureus
Crytococcus
Streptococcus pneumoniae
E.
129.
A. *
B.
C.
D.
E.
130.
A. *
B.
C.
D.
E.
131.
A. *
B.
C.
D.
E.
132.
A. *
B.
Mycobacterium avium
A 50-year-old man is evaluated in the emergency department because of fever, a nonproductive
cough, and a 2-day history of myalgia and headache. He has also had some nausea and diarrhea. He
is a heavy smoker. On physical examination, he is slightly disoriented. Temperature is 38.9 0C, pulse
rate is 110/min, respiration rate is 32/min. Chest radiograph shows fluffy infiltrates to the right upper
and lower lobes. Results of laboratory testing show serum sodium of 128 meq/L, blood urea nitrogen
of 42 mg/dL, serum creatinine of 2.2 mg/dL, and serum creatine kinase of 250 U/L. Which one of the
following is best next step in the management of this patient’s pneumonia?
Initiate empiric antibiotic therapy for Legionella
Order direct fluorescent antibody testing of the sputum for Legionella
Order serologic testing for Legionella
Send a urine specimen for measurement of Legionella antigen
All of the above
A 50-yr-old man presents with shortness of breath and dry cough. CXR shows widespread pulmonary
shadowing. He takes Azathioprine for resistant rheumatoid arthritis. Choose the most appropriate
treatment.
Co-trimoxazole
Erythromycm
Tetracycline
Flucoxacillin
Isoniazid
A 52-year-old man is evaluated because of a 2-month history of nonproductive cough, myalgias, and
low-grade fever. When his illness began, a chest radiograph showed bilateral alveolar infiltrates, and
a presumptive diagnosis of community-acquired pneumonia was made. He was treated with oral
azithromycin without effect, followed by a 10-day course of levofloxacin, also without effect. During
the course of his illness he has lost 4.5 kg without significant anorexia. He is a lifetime nonsmoker
and works as an office manager. He has no pets and no unusual hobbies. On physical examination,
his vitals signs are normal, except for a respiration rate of 22/min. He is in mild respiratory distress
on exertion. On examination of the chest, bilateral crackles are audible, without wheezing. Chest
radiograph shows bilateral alveolar infiltrates, which are changed in location from those seen on his
original radiographs. Pulmonary function tests show forced expiratory volume in 1 sec (FEV1) 75%
of predicted, forced vital capacity (FVC) 72% of predicted, total lung capacity 80% of predicted, and
diffusing lung capacity for carbon monoxide (DL CO) 65% of predicted. Arterial blood gas values,
with the patient breathing room air, are PO2 62 mm Hg, PCO2 42 mm Hg, and pH 7.39. Which of
the following is the most likely diagnosis?
Cryptogenic organizing pneumonitis
Hypersensitivity pneumonitis
Resistant pneumococcal pneumonia
Chlamydia pneumonia
Bronchoalveolar cell carcinoma
A 60-year-old man with chronic obstructive pulmonary disease is evaluated because of persistent
cough and sputum production. He reports no fevers or weight loss. He has had increased sputum
production over the past 6 to 9 months and moderately severe reduction in forced expiratory volume
in 1 sec (FEV1 55% of predicted). His symptoms have been unresponsive to antibiotics. He has an
80-pack-year history of cigarette smoking. Chest radiograph reveals multiple small nodules in the left
upper lung zone without infiltrate. Serologic testing for HIV is negative. Sputum cultures are
negative for bacteria. A test for acid-fast bacillus is negative, but culture grows Mycobacterium
avium-intracellulare with 1+ growth. Which of the following is the best next step in this patient’s
Order
a sputum culture for Mycobacterium avium-intracellulare
management?
Treat him for Mycobacterium avium-intracellulare without further testing
C.
A. *
Perform bronchoscopy with bronchioalveolar lavage and transbronchial biopsy
Order a HRCT scan of the chest
All of the above
A 62-year-old woman with moderate emphysema is evaluated during a routine visit. She has chronic
dyspnea on exertion but has no cough or sputum production. She uses supplemental oxygen, 2 L/min,
when sleeping and on exertion. She currently uses albuterol and ipratropium four times per day, and
salmeterol and theophylline twice per day. She is currently enrolled in a pulmonary rehabilitation
program and is concerned about “catching a cold” from other people enrolled in the pulmonary
rehabilitation program. What is the best advice for this patient?
Practice good hand washing, attempt to avoid close prolonged contact with ill persons, and take
pneumococcal and annual influenza vaccine
Avoid any social functions where there will be large crowds
Discontinue attendance at the pulmonary rehabilitation program
Take a daily antibiotic (long-term suppressive antibiotic therapy) to prevent pneumonia
All of the above
A 65-yr-old man currently undergoing chemotherapy of chronic leukaemia has felt unwell with fever
and unproductive cough for 2 weeks despite treatment with broad-spectrum IV antibiotics. The CXR
shows an enlarging right sided midzone consolidation. Which infection is the most possible cause of
his state?
Fungi
Streptococcus pneumoniae
Mycoplasma pneumoniae
Pneumocystis carinii
Legionella pneumoniae
A 67-year-old woman is evaluated in the emergency department because of a 2-day history of fever
and a cough productive of purulent sputum. She is intubated for hypoxic respiratory failure due to
pneumonia and hospitalized in the intensive care unit. Her medical history is remarkable only for
hypertension for which she takes a calcium channel blocker. She has never worked outside the home,
has no pets, and has not traveled recently. Which of the following would be appropriate initial
antibiotic therapy?
A second-generation cephalosporin and macrolide
A second-generation cephalosporin
A third-generation cephalosporin, aminoglycoside, and macrolide
An antipseudomonal ?-lactam/?-lactamase combination and fluoroquinolone
All of the above
A 70-year-old man is ready to be discharged in October from the hospital after treatment of an
exacerbation of his chronic obstructive pulmonary disease. His only other medical problems include
stable angina and hypertension. He has normal renal function and is well nourished. He quit smoking
2 years ago. He received pneumonia vaccine 2 years ago and influenza vaccine 1 year ago. Which of
the following is true of his vaccination status?
He should receive influenza vaccine
B.
C.
D.
E.
He should receive pneumonia vaccine and influenza vaccine
He should receive pneumonia vaccine
He should receive neither pneumonia vaccine nor influenza vaccine
Nothing of the above
D.
E.
133.
A. *
B.
C.
D.
E.
134.
A. *
B.
C.
D.
E.
135.
A. *
B.
C.
D.
E.
136.
137.
A. *
B.
C.
D.
E.
138.
A. *
B.
C.
D.
E.
139.
A. *
B.
C.
D.
E.
140.
A.
B. *
C.
D.
E.
141.
A. *
B.
C.
A 70-year-old woman resident of a nursing home is evaluated in the emergency department because
of decreasing mental status and hypothermia. She has a history of stroke and is currently taking only
aspirin. She has been able to eat on her own and there have been no witnessed aspirations. She has
not been treated recently with antibiotics. Her leukocyte count is 12,000/L, and her hemoglobin is
120 g/L. Serum electrolytes are within normal limits and she has mild chronic renal insufficiency.
Chest radiograph shows a small interstitial infiltrate in the right lower lung field. She receives
traditional empiric treatment for community-acquired pneumonia. Therapy for which of the
following should also be considered?
Enteric gram-negative organisms
Pseudomonas aeruginosa
Anaerobic bacteria
Aspergillus fumigatus
Mycobacterium tuberculosis
A 72-year-old woman with a history of rheumatoid arthritis is evaluated because of persistent cough,
purulent sputum production, and occasional scant hemoptysis. She reports no fever, but says she has
difficulty gaining weight. She has never smoked cigarettes. She is hospitalized because of tachypnea
and hypoxia. Spirometry shows moderately severe obstructive impairment. Chest radiograph shows
tramlines without infiltrate. Gram stain of the sputum shows numerous leukocytes but no bacteria;
culture results are pending. Previous IgE levels were normal. There is no eosinophilia. An immediate
skin test for Aspergillus is negative. A tuberculin skin test performed last year was nonreactive.
Which of the following is appropriate therapy for this patient?
Intravenous fluoroquinolone and aminoglycoside
A ?-lactam/?-lactamase combination
Itraconazole
A new-generation macrolide
All of the above
A businessman, had a rest during summer on Cyprus in 5-stars hotel. After 3 weeks of rest suddenly,
without any cause, has got severe headache, pain in muscles, joints, dry cough, moderate dyspnea,
pain in a thorax. X-ray: infiltration of lung tissue. In a blood test: absolute lymphopenia with
moderate leucocytosis. Most possible etiological factor of pneumonia:
Legionella
Mycoplasma
Pneumocysts
Enteroviruses
Pneumococcus
A patient who is 2 days postoperative from a bowel resection tells her physician that she is having a
hard time “catching her breath,” feels nauseated, and has chest pains when she inhales. The physician
suspects that she is having a pulmonary embolism. What intervention should the physician perform
before notifying the physician?
Increase the IV flow rate
Apply oxygen by mask or nasal cannula at 5 L/min
Assess the chest and axillary area for the presence of petechiae
Place the patient in shock position, with her head and neck flat and her legs elevated
Non of above
A pet shop owner presents with high swinging fever, cough and malaise. He has scanty rose spots
over his abdomen. CXR reveals diffuse pneumonia. Which infection is the most possible cause of his
state?
Chlamydia psittacci
Coxiella burnetti
Staphylococcus aureus
D.
A. *
Pneumocystis carinii
Aspergillosis
A previously healthy 18-yr-old girl has had influenza for the last 2 weeks. She is deteriorating and
has a swinging fever. She is coughing up copious purulent sputum. CXR shows cavitating lesions.
Which infection is the most possible cause of his state?
Staphylococcus aureus
Streptococcus pneumoniae
Mycoplasma pneumoniae
Pneumocystis carinii
Legionella pneumoniae
During the epidemic of influenza patient G., 59 y.o., after decreasing of fever noticed pain appeared
in a thorax, cough with yellow-green sputum (amount-100 ml a day), sometimes with some blood.
Objectively: breathing rate - 36/min. In lungs from the right side lower scapula there is dull sound
during percussion, hard breathing, and moist rales. Blood test: L - 18,6х109/л, ESR -64 mm/h.
Analysis of sputum: L -80-100 , Er - 40-50, elastic fibres, cocci. X-ray: rhadicis are enlarged, from
the right side lower lobe is heterogeneously infiltrated with two lighter areas. What is the most
possible previous diagnosis?
Right-side pneumonia with abscesses
Peripheral cancer
Infiltrative tuberculosis in the phase of disintegration
Exudative pleurisy
Infarction-pneumonia
Female B., 44 years old, complains on cough with mucous sputum, increase of temperature to 39 °С,
weakness, dyspnea, sweating. Breathing rate - 26/min., skin is moist. Below left scapula there is
shortening of percussion sound. Breathing during auscultation is weaker, moist rales. Blood test: L 11х109/l, ESR - 29 mm/h. Your previous diagnosis?
Left-side lower lobe pneumonia
Gangrene of lungs
Left-side exudative pleurisy
Cancer of left side lower lobe
Pulmonary abscess
Female patient K., 46 years old, after decreasing of fever after flu noticed pain appeared in a thorax,
cough with yellow-green sputum (amount-150 ml a day), sometimes with some blood. Objectively:
breathing rate - 36/min. In lungs from the right side lower scapula there is dull sound during
percussion, hard breathing, and moist rales. Blood test: L - 18,6х109/l, ESR -64 mm/h. Analysis of
sputum: L -80-100 , Er - 40-50, elastic fibres, cocci. X-ray: rhadicis are enlarged, from the right side
lower lobe is heterogeneously infiltrated with two lighter areas. What is the most possible previous
diagnosis? pneumonia with abscesses
Right-side
Peripheral cancer
Infiltrative tuberculosis in the phase of disintegration
Exudative pleurisy
Infarction-pneumonia
Female, 34 years old, has an increase of body temperature to 38 °С, cough with purulent sputum,
weakness, dyspnea, pain in a thorax during breathing. During percussion there is shortening of sound
in the lower part of left lung, during auscultation – moist rales. What method of investigation is the
decisive one to confirm diagnosis?
Chest X-ray examination
B.
Bacteriological analysis of sputum
E.
142.
A. *
B.
C.
D.
E.
143.
A. *
B.
C.
D.
E.
144.
A. *
B.
C.
D.
E.
145.
A. *
B.
C.
D.
E.
146.
C.
D.
E.
147.
A. *
B.
C.
D.
E.
148.
A. *
B.
C.
D.
E.
149.
A. *
B.
C.
D.
E.
150.
A. *
B.
C.
D.
E.
151.
A. *
B.
C.
D.
E.
Spirometry
Pneumotachometry
Bronchography
Male patient F., 48 years old, during a week was at home with diagnosis of respiratory viral infection.
Doctor noticed complaints on cough with small amount of mucus-purulent sputum, weakness.
Objectively: condition is relatively satisfactory. T - 37,2 °С. Breathing rate - 18/min, pulse - 80/min,
BP - 110/70. In lungs there is vesicular breathing, with a hard tint, single dry wheezes. Tones of heart
are a little dull, rhythm is correct. What is the treatment tactic?
To prescribe antibacterial therapy
To stay at home for some more days
To go to work
To send patient to pulmonologist
To hospitalize patient to the pulmonological department
Male patient G., 56 years old, complaints on permanent pain in a thorax which disturbs for last 2
months. Pain is not connected with breathing. There is also cough with particles of blood in sputum.
Weakness, fatigue are present. On the X-ray of thorax in the lower lobe of right lung there is
spherical shadow, size 4x6 cm, related to the lungs rhadicis. What is the most possible diagnosis?
Perypheral lung cancer
Tuberculoma
Metastasis
Pulmonary abscess
Pneumonia
Man 40 y.o., is ill with attacks of cough with yellow-brown sputum, pain in a right side, related to the
deep breathing, sweating. He is ill for 6 days, after overcooling. Used aspirin. Objectively: T - 39,6
°С, breathing rate - 26/min, pulse - 110/min, BP -110/70. In lower part of right lung - moist loud
rales. Chest X-ray: in right lower lobe there is massive unhomogeneous infiltration with lighter areas,
sinus is not changed. What complication of disease is the most probable?
Abscesses
Dry pleurisy
Empiema of pleura
Spontaneous pneumothorax
Pulmonary athelectasis
On return to university, a 20-yr-old student presented with the onset of fever, malaise and a dry
cough. The student health service gave him Amoxycilline. After a week he felt no better and his CXR
showed patchy bilateral consolidation. Which infection is the most possible cause of his state?
Mycoplasma pneumoniae
Streptococcus pneumoniae
Pneumocystis carinii
Fungi
Legionella pneumoniae
Patient B., 26 years old, who used to drink alcohol, has right-side lower-lobe pneumonia. On the
chest X-ray film there are infiltrative changes from the right side – in S 6 and S 10. He has no
diseases of the respiratory tract before. From which medicine is it better to begin antibacterial
therapy?
Amoxyclav
Biseptol
Ofloxacin
Tetracyclin
Ceporin
152.
A. *
B.
C.
D.
E.
153.
A. *
B.
C.
D.
E.
154.
A. *
B.
C.
D.
E.
155.
A. *
B.
C.
D.
E.
156.
A.
B.
C.
D.
E. *
157.
A. *
Patient K., 27 years old, has dry cough and pain, related to breathing, in the right half of thorax, fever
- to 39,5 °С. In the right lung lower from scapula there is dullness during percussion, bronchial
breathing. What is the most possible diagnosis for a patient?
Right-side lower lobe pneumonia
An abscess of lower lobe of right lung
Bronchiectasis
An athelectasis of lower lobe of right lung
Exudative pleurisy
Patient F., 38 years old, complaints on increase of temperature to 37,9 °С, cough with small amount
of mucus-purulent sputum, pain in right part. He is ill for 5 days, after overcooling. Objectively:
acrocyanosis. Pulse - 96/min. BP - 120/80. From the right side below scapula there is increased vocal
fremitus, shortened percussion sound, moist rales. What is the most possible diagnosis?
Community-acquired right side lobar pneumonia
Right lung athelectasis
Right-side exudative pleurisy
Right lung athelectasis
Infiltrative tuberculosis
Patient of 53 y.o., complaints on cough with mucous sputum, increase of temperature to 39 °С,
weakness, dyspnea, sweating. Breathing rate - 26/min, skin is moist. Below left scapula there is
shortening of percussion sound. Breathing during auscultation is weaker, moist rales. Blood test: L 11х109/l, ESR - 29 mm/h. Your previous diagnosis?
Left-side lower lobe pneumonia
Gangrene of lungs
Left-side exudative pleurisy
Cancer of left side lower lobe
Pulmonary abscess
Patient of 56 years old, complaints on permanent pain in a thorax which disturbs for last 2 months.
Pain is not connected with breathing. There is also cough with particles of blood in sputum.
Weakness, fatigue are present. On the X-ray of thorax in the lower lobe of right lung there is
spherical shadow, size 4x6 sm, related to the lungs rhadicis. What is the most possible diagnosis?
Perypheral lung cancer
Tuberculoma
Metastasis
Pulmonary abscess
Pneumonia
Patient G., 36 years, works on a poultry factory. Her emergency hospitalized with acute attack of
dyspnoea. During observation was diagnosed bronchial asthma. What additional methods of research
must be conducted above all things for confirmation of professional genesis of bronchial asthma?
roentgenologic research of breathing organs
professional route of patient
sanitary-hygienic characteristic of work conditions
research of function of the external breathing
allergic and immunologic tests
Patient G., 59 y.o., after decreasing of fever after flu noticed pain appeared in a thorax, cough with
yellow-green sputum (amount-150 ml a day), sometimes with some blood. Objectively: breathing
rate - 36/min. In lungs from the right side lower scapula there is dull sound during percussion, hard
breathing, and moist rales. Blood test: L - 18,6х109/l, ESR -64 mm/h. Analysis of sputum: L -80-100
, Er - 40-50, elastic fibres, cocci. X-ray: rhadicis are enlarged, from the right side lower lobe is
heterogeneously infiltrated with two lighter areas. What is the most possible previous diagnosis?
Right-side pneumonia with abscesses
B.
C.
D.
E.
158.
A. *
B.
C.
D.
E.
159.
A. *
B.
C.
D.
E.
160.
A. *
B.
C.
D.
E.
161.
A. *
B.
C.
D.
E.
162.
A. *
Peripheral cancer
Infiltrative tuberculosis in the phase of disintegration
Exudative pleurisy
Infarction-pneumonia
Patient I., a 50 years old man is evaluated in the emergency department because of fever, a
nonproductive cough, and a 2-day history of myalgia and headache. He has also had some nausea and
diarrhea. He is a heavy smoker. On physical examination, he is slightly disoriented. Temperature is
38.9 C, pulse rate is 110/min, respiratory rate is 32/min. Chest radiograph shows fluffy infiltrates to
the right upper and lower lobes. Results of laboratory testing show serum sodium of 128 meq/L,
blood urea nitrogen of 42 mg/dL, serum creatinine of 2.2 mg/dL, and serum creatine kinase of 250
U/L. Which one of the following is best next step in the management of this patient’s pneumonia?
Initiate empiric antibiotic therapy for Legionella
Order direct fluorescent antibody testing of the sputum for Legionella
Order serologic testing for Legionella
Send a urine specimen for measurement of Legionella antigen
All of the above
Patient K. 38 y.o., who is in a hospital, after week noticed temperature increased to 39 0С, cough
appeared with “ferruginous” sputum, pain in a thorax, related to the act of breathing, breathing rate is
26 per minute. On X-ray there is infiltration in the lower lobe of the left lung. What medicine is
useful?
Cephalosporin III generation
Tetracyclin
Penicillin
Erythromycin
Streptomycin
Patient M., 39 years old, is ill with attacks of cough with yellow-brown sputum, pain in a right side,
related to the deep breathing, sweating. He is ill for 6 days, after overcooling. Used aspirin.
Objectively: T - 39,6 °С, breathing rate - 26/min., pulse - 110/min., BP -110/70. In lower part of right
lung - moist loud rales. X-ray: in right lower lobe there is massive unhomogeneous infiltration with
lighter areas, sinus is not changed. What complication of disease is the most possible?
Abscesses
Dry pleurisy
Empiema of pleura
Spontaneous pneumothorax
Pulmonary athelectasis
Patient N., 46 years old, was admitted into the surgical department and operated because of
appendicitis. After 4 days appeared she developed chills, cough, dyspnea, fever 38,5 °С, leucocytosis
with shift of leucocyte formula to the left. On chest X-ray there is infiltration of lower right lobe.
What is the diagnosis?
Nosocomial pneumonia
Pulmonary abscess
Infarction pneumonia
Community-acquired pneumonia
Tuberculosis
Patient O., 29-yr-old male prostitute, has felt generally unwell for 2 months with some weight loss.
Over the last 3 weeks he has noticed a dry cough with increasing breathlessness. Two courses of
antibiotics from the GP have produced no improvement. The CXR shows bilateral interstitial
infiltrates. What is the most possible etiology of disease?
Pneumocystis carinii
B.
C.
D.
E.
163.
A. *
B.
C.
D.
E.
164.
A. *
B.
C.
D.
E.
165.
A. *
B.
C.
D.
E.
166.
A. *
B.
C.
D.
E.
167.
A. *
B.
C.
Streptococcus pneumoniae
Mycoplasma pneumoniae
Fungi
Legionella pneumoniae
Patient of 51 y.o., complaints on weakness, dyspnea, pain in the left half of thorax, permanent cough
with viscid sputum, in which particles of blood are sometimes determined. For the last 3 months he
lost 5 kg of body weight. On the X-ray of lungs there is total homogeneous shade is determined from
the left side. Organs of mediastinum are displaced to the left. What diagnosis is possible?
Lung athelectasis
Lungs gangrene
Total exudative pleurisy
Pneumonia
Empyema of pleura
A 38 years old patient, who drunk a lot of alcohol, has severe pneumonia. His condition got worse,
the temperature of body rose to 39-40 °С, an unpleasant smell appeared from a mouth, increased
amount of purulent sputum; increased ESR and amount of band leucocytes. On the X-ray - in the
lower lobe of right lung there is massive infiltration with light area in a center. What complication is
it necessary to suspect?
Acute pulmonary abscess
Bronchiectasis
Infarction-pneumonia
Gangrene of lungs
Empyema of pleura
Patient of 43, complaints on cough with small amount of colourless sputum, pain in the right half of
thorax during breathing, shortness of breath, increase of temperature to 39 °С. Felt ill rapidly. Used
aspirin. Objectively: gerpes on lips. In lower lobe of right lung there is dull percussion sound,
bronchial breathing. X-ray: there is homogeneous infiltration of right lower lobe. What is the most
possible etiology of pneumonia?
Pneumococcus
Staphylococcus
Mycoplasma
Legionella
Klebsiella
Patient P., a young male homosexual with Kaposi's sarcoma, complains of increasing breathlessness
and a dry cough. He has a 3-day history of shivering, general malaise and productive cough. The
X-ray shows right lower lobe consolidation. What is the most possible etiology of disease?
Pneumocystis carinii
Mycoplasma tuberculosis
Haemophilus influenzae
Chlamydia trachomatis
Klebsiella pneumoniae
Patient Q., 37 years old, was admitted into the surgical department and operated because of
appendicitis. After 4 days appeared she developed chills, cough, dyspnea, fever 38,5 °С, leucocytosis
with shift of leucocyte formula to the left. On chest X-ray there is infiltration of lower right lobe.
What is the diagnosis?
Nosocomial pneumonia
Pulmonary abscess
Infarction pneumonia
D.
E.
168.
A. *
B.
C.
D.
E.
169.
A. *
B.
C.
D.
E.
170.
A. *
B.
C.
D.
E.
171.
A. *
B.
C.
D.
E.
172.
A. *
B.
Community-acquired pneumonia
Tuberculosis
Patient S., a 25-year-old male has just returned from holiday abroad, presents with flu-like illness,
headaches, high fever. Prior to this, he had complained of abdominal pain, vomiting, diarrhea
associated with blood per rectum. What is the most possible etiology of disease?
Legionella pneumoniae
Streptococcus pneumoniae
Mycoplasma pneumoniae
Pneumocystis carinii
Fungi
Patient V., a 24 years old barman, presents with a dry cough of sudden onset. He complains of a chest
pain and rusty sputum. He also has a very high fever, rapid breathing, cyanosis and crepitations.
Pneumonia was suspected. What is the most nesessary method of investigation?
Chest X-ray
Spirography
Analysis of sputum
General blood analysis
General urine analysis
Patient W., 67 years old, during the epidemic of influenza after decreasing of fever noticed pain that
appeared in a thorax, cough with yellow-green sputum (amount-100 ml a day), sometimes with some
blood. Objectively: breathing rate - 36/min. In lungs from the right side lower scapula there is dull
sound during percussion, hard breathing, and moist rales. Blood test: L - 18,6х109/l, ESR -64 mm/h.
Analysis of sputum: L -80-100 , Er - 40-50, elastic fibres, cocci. X-ray: rhadicis are enlarged, from
the right side lower lobe is heterogeneously infiltrated with two lighter areas. What is the most
possible previous diagnosis?
Right-side pneumonia with abscesses
Peripheral cancer
Infiltrative tuberculosis in the phase of disintegration
Exudative pleurisy
Infarction-pneumonia
Patient X., 55 years old, was admitted to the hospital recently. He complaints on cough with very
small amount of mucous-purulent sputum, significant weakness, increased temperature, which is
accompanied with chill, dizziness. Objectively: t - 38°С. Breathing rate - 22/min. Heart rate - 90/min,
BP - 110/70. From the right side below scapula the vocal fremitus is increased, percussion sound is
shortened, vesicular breathing is weakened, small amount of moist rales. Tones of heart are dull,
rhythm is correct, moderate tachycardia. Doctor suspected pneumonia. The presence of what
syndrome let to suspect such diagnosis?
Pulmonary tissue infiltration
Intoxication
Inflammation
Bronchial obstruction
Respiratory insufficiency
Patient Z., a 33-yr-old car mechanic is brought to casualty by his girlfriend. She describes a 2-day
history of rigors, sweats and intermittent confusion. On examination he is agitated, sweaty and
pyrexial with 38.6° C. He is hyperventilating and cyanosed despite receiving O2 by face mask. There
is dullness to percussion and bronchial breathing at the left lung base. What method of investigation
is necessary?
Chest
X-ray
Spiral CT with contrast
C.
D.
E.
173.
A. *
B.
C.
D.
E.
174.
A. *
B.
C.
D.
E.
175.
A. *
B.
C.
D.
E.
176.
A. *
B.
C.
D.
E.
177.
A.
B.
C.
D. *
Arterial blood gases
Blood count and film
Urea and electrolytes
Previously healthy 28-year-old man is evaluated in the emergency department because of fever,
productive cough, and shortness of breath. His temperature is 40 C, pulse is 120/min, respiration rate
is 32/min, and blood pressure is 100/70 mm Hg. Measurement of arterial blood gases with the patient
breathing room air shows PO2 of 55 mm Hg, PCO2 of 30 mm Hg, pH of 7.41. Chest radiograph
reveals bilateral alveolar infiltrates with no effusions. Gram stain of the sputum reveals gram-positive
diplococci. Which of the following is the most appropriate for this patient?
Hospitalize him
Treat him as an outpatient with oral therapy
Treat him as an outpatient with intravenous therapy
Hospitalize the patient in the intensive care unit
All of the above
Previously healthy 32-yr-old woman presents with general malaise, severe cough and breathlessness,
which has not improved with a 7 day-long course of Amoxycillin. There is nothing significant to find
on examination. The X-ray shows patchy shadowing throughout the lung fields. The blood film
shows clumping of red cells with suggestion of cold agglutinins. What is the most possible etiology
of disease? pneumoniae
Mycoplasma
Legionella pneumonia
Haemophilus influenzae
Chlamydia trachomatis
Klebsiella pneumoniae
Female, 34 years old, has an increase of body temperature to 38 °С, cough with purulent sputum,
weakness, dyspnea, pain in a thorax during breathing. During percussion there is shortening of sound
in the lower part of left lung, during auscultation – moist rales. What method of investigation is the
decisive one to confirm diagnosis?
X-ray examination
Bacteriological analysis of sputum
Spirometry
Pneumotachometry
Bronchography
Female B., 44 years old, complains on cough with mucous sputum, increase of temperature to 39 °С,
weakness, dyspnea, sweating. Breathing rate - 26/min, skin is moist. Below left scapula there is
shortening of percussion sound. Breathing during auscultation is weakened, moist rales. Blood test: L
- 11х109/l, ESR - 29 mm/h. Your previous diagnosis?
Left-side lower lobe pneumonia
Gangrene of lungs
Left-side exudative pleurisy
Cancer of left side lower lobe
Pulmonary abscess
The 60-year-old patient tells you that she smoked three packs of cigarettes per day since she was 15
years old, until she was 40, and then smoked two packs per day. How many pack-years should you
record in the patient's history?
45
80
90
115
E.
178.
A.
B. *
C.
D.
E.
179.
A.
B. *
C.
D.
E.
180.
A.
B. *
C.
D.
E.
181.
A.
B. *
C.
D.
E.
182.
A.
B.
C.
D. *
E.
Non of above
The 82-year-old patient has a pulmonary infection. Which action addresses the age-related change of
increased vascular resistance to blood flow through pulmonary vasculature in this patient?
Encouraging the patient to turn, cough, and deep breathe every hour
Assessing the patient's level of consciousness
Raising the head of the bed
Humidifying the oxygen
Non of above
The patient diagnosed with moderate stage COPD says there is no sense in stopping smoking now
because the damage is done. Which response is the best rationale for encouraging this patient to stop
smoking?
“The damage will be reversed.”
“The COPD will progress more slowly.”
“Your risk for asthma development, which would further reduce your lung function, will be
decreased.”
“You
will be less likely to lose excessive amounts of weight and will have a more normal
appearance.”
Non
of above
The patient has broken ribs that penetrated through the skin as a result of a motor vehicle crash 3
days ago. The patient now complains of increased pain, shortness of breath, and fever. Which
assessment finding alerts the physician to the possibility of a pleural effusion and empyema?
Wheezing on exhalation on the side with the broken ribs
Absence of fremitus at and below the site of injury
Crepitus of the skin around the site of injury
Absence of gastric motility
Non of above
The patient has severe nasal congestion, headache, and sneezing but no rhinorrhea, watery eyes, sore
throat, or fever. Which statement made by the patient alerts the physician to the possibility of rhinitis
medicamentosa?
“I have been taking two aspirins every 6 hours for this headache.”
“My nose doesn't stay open even though I'm using nasal spray every hour.”
“I have been taking a lot of vitamin C this year to keep from getting so many colds.”
“The only way I can get to sleep with this nasal congestion is by taking an over-the-counter
antihistamine at night.”
Non of above
The patient is 34 years old and has been diagnosed with COPD as a result of being homozygous for a
mutation of the alpha1-antitrypsin (AAT) gene alleles. His wife has two normal AAT gene alleles.
He is concerned that his two children may develop this problem. What is your best response?
“Because neither of your parents have COPD and your wife does not have the abnormal gene alleles,
your children will not be affected.”
“Because your wife is not affected nor is or a carrier, your children will have normal levels of AAT
and their risk is the same as for the general population.”
“Because you have the mutations and your wife does not, your son will be at an increased risk for
developing COPD but your daughter will only be a carrier.”
“Because both of your AAT gene alleles are mutated, your children will each have one abnormal
gene and their risk for COPD is only increased if they smoke or are chronically exposed to other
precipitating factors.”
Non of above
183.
A. *
B.
C.
D.
E.
184.
A. *
B.
C.
D.
E.
185.
A. *
B.
C.
D.
E.
186.
A. *
B.
C.
D.
E.
187.
The patient with hospital-acquired (nosocomial) pneumonia caused by a bacterial infection with a
gram-negative microorganism is receiving treatment with intravenous amikacin (Amikin). In addition
to frequent respiratory assessment, what other assessment should the physician routinely perform to
identify a common complication of this medication?
Monitor urine output every shift
Perform neuro checks every 2 hours
Examine the stool and vomitus for the presence of blood
Monitor the complete white blood cell count and differential daily
Non of above
A 35-year-old man is evaluated because of a 2-week history of low-grade fevers, fatigue, cough,
pleuritic chest pain, and increasing dyspnea on exertion. He is a construction worker and is having
difficulty performing his usual tasks. He has a 10-pack-year history of cigarette smoking. On
physical examination, he has right chest pain but no respiratory distress at rest. Temperature is 38.2
C, pulse rate is 112/min and regular, and respiration rate is 20/min. There is evidence of a right
pleural effusion and no other abnormalities. Peripheral blood leukocyte count is 9,000/L, with 80%
neutrophils and 15% lymphocytes. Liver function test results are normal. Chest radiograph shows a
moderate right pleural effusion with minimal contralateral shift and no parenchymal infiltrates.
Thoracentesis yields minimally turbid yellow fluid with results as follows: Pleural fluid nucleated
cell count 3000/L with 5% neutrophils, 85% lymphocytes, and 1% macrophages Pleural fluid total
protein 5.2 g/dL Pleural fluid serum lactate dehydrogenase 230 U/L Pleural fluid glucose 80 mg/dL
Pleural fluid pH 7.36 Pleural fluid Gram and acid-fast bacilli stains are negative. Tuberculin skin test
is negative. Cytologic evaluation for malignant cells is negative. What is the most likely diagnosis?
Tubercular pleurisy
Lung cancer
Parapneumonic effusion
Pulmonary embolism
Benign asbestos pleural effusion
A 37-yr-old man who has had recurrent chest infections since a serious bout of influenza 3-yr ago
presents with chronic productive cough. His sputum is tenacious and blood stained. On auscultation
you find crackling. What treatment is necessary?
Postural drainage
Surgical excision
Prednisolone
Ipratropium
Pleurectomy
A 38 years old patient, who drunk a lot of alcohol, has severe pneumonia. His condition was
worsened, the temperature of body rose to 39-40 °С, an unpleasant smell appeared from a mouth,
increased amount of purulent sputum; increased ESR and amount of band leucocytes. On the X-ray in the lower lobe of right lung there is massive infiltration with light area in a center. What
complication is it necessary to suspect?
Acute pulmonary abscess
Bronchiectasis
Infarction-pneumonia
Gangrene of lungs
Empyema of pleura
A 48 years old patient, complaints on weakness, dyspnea, pain in the left half of thorax, permanent
cough with viscid sputum, in which particles of blood are sometimes determined. For the last 3
months he lost 5 kg of body mass. On the X-ray of lungs there is total homogeneous shade
determined from the left side. Organs of mediastinum are displaced to the left. What diagnosis is
possible?
A. *
B.
C.
D.
E.
188.
A. *
B.
C.
D.
E.
189.
A. *
B.
C.
D.
E.
Lung athelectasis
Lung gangrene
Total exudative pleurisy
Pneumonia
Empyema of pleura
A 56-year-old construction worker, a heavy smoker, sustains severe trauma to his left chest. Chest
pain is severe for several minutes but subsides over the next hour. Because the chest pain does not
resolve completely, he is evaluated in the emergency department 2 hours later, where results of a
chest radiograph and complete blood count with differential are normal. The next day, he leaves on a
week-long vacation to South America. During that time, he has intermittent chest discomfort and
gradually increasing dyspnea with exertion. Upon returning home, he sees his physician because of
dyspnea. Chest radiograph shows a large left-sided pleural effusion with minimal contralateral shift.
At thoracentesis, 500 cc of brownish-colored fluid is removed and analyzed with the following
results. Pleural fluid nucleated cell count 4000/mL with 10% neutrophils, 30% lymphocytes, 15%
macrophages, and 45% eosinophils Pleural fluid hematocrit 10% Pleural fluid total protein 4 g/dL
Pleural fluid serum lactate dehydrogenase 200 UIL Pleural fluid glucose 80 mg/dL Pleural fluid pH
7.35 Cytology test results are negative. Pain medication is prescribed for the patient. When he returns
14 days later, complete blood count shows a leukocyte count of 9000/?L with 20% eosinophils and
chest radiograph shows that the pleural effusion has decreased substantially. Which of the following
Post-traumatic
hemothorax
is the most likely
diagnosis?
Benign asbestos pleural effusion
Paragonimiasis
Lung cancer
Pulmonary infarction
A 60-year-old man is evaluated because of a 6-week history of progressive dyspnea on exertion,
fatigue, a decrease in appetite, and a weight loss of 1.8 kg. He has a 30-pack-year history of cigarette
smoking and drinks two or three cocktails every evening. He has no gastrointestinal complaints and
no history of a febrile illness. On physical examination, he is afebrile with normal vital signs. The
only abnormalities noted on chest examination are findings compatible with a right pleural effusion.
Chest radiograph confirms a pleural effusion occupying 40% of the right hemithorax without
evidence of loculation. There are no obvious parenchymal lesions and no mediastinal adenopathy.
Results of pleural fluid analysis are as follows. Pleural fluid nucleated cell count 2800/?L with 10%
neutrophils, 50% lymphocytes, 30% macrophages, and 10% mesothelial cells Pleural fluid total
protein 3.8 g/dL (pleural fluid/serum ratio 0.60) Pleural fluid serum lactate dehydrogenase 210 U/L
(ratio of pleural fluid to upper limits of normal serum lactate dehydrogenase 0.72) Pleural fluid
amylase 30 mg/dL (pleural fluid/serum ratio 0.5) Pleural fluid glucose 50 mg/dL Pleural fluid pH
7.26 Which of the following is the most likely diagnosis?
Malignant effusion
Complicated parapneumonic effusion
Esophageal rupture
Rheumatoid pleurisy
Acute pancreatitis
190.
A. *
B.
C.
D.
E.
191.
A. *
B.
C.
D.
E.
192.
A. *
B.
C.
D.
E.
193.
A. *
B.
C.
D.
E.
A 60-year-old man with a history of alcohol abuse is evaluated because of the insidious onset of
dyspnea over the course of 6 weeks. He has no cough, chest or abdominal pain, or hemoptysis. He
smoked 1.5 packs of cigarettes per day for 25 years and stopped smoking 4 years ago. He has a
moderate-sized right pleural effusion. Chest radiograph shows the effusion with minimal
contralateral mediastinal shift and is otherwise normal. Pleural fluid analysis shows clear yellow fluid
with 500 nucleated cells/L, 10% neutrophils, 25% lymphocytes, 60% macrophages, and 5%
mesothelial cells. Pleural fluid values are as follows: Total protein 1.1 g/dL Serum lactate
dehydrogenase 4l U/L Serum amylase 20 U/L Glucose 100 mg/dL pH 7.45 Ratio of pleural fluid to
serum (PF/S) total protein 0.2 Ratio of pleural fluid to serum lactate dehydrogenase (upper limits of
normal) 0.35 Other laboratory results show a serum albumin of 2.4 g/dL, an INR of 1 .5, and a
normal urinalysis. Electrocardiogram is normal. Which of the following is the most likely diagnosis?
Hepatic hydrothorax
Congestive heart failure
Chronic pancreatitis
Lung cancer
Nephrotic syndrome
A 72-year-old woman is evaluated because of morning headaches and swelling in the lower
extremities that worsens as the day progresses. She is able to sleep supine, using one pillow at night,
and notices shortness of breath walking distances greater than 20 feet. On physical examination,
significant findings include diminished breath sounds, distant heart sounds, with pulmonic valve
component equal in intensity to aortic valve component, and paradoxical splitting of the S2.
Laboratory finding include a Pa O2of 59mm Hg, Pa CO2 of 44 mm Hg, and pH of 7.41.
Electrocardiogram shows right ventricular hypertrophy with cor pulmonale and right axis deviation.
Which of the following is the best rationale for long-term oxygen therapy for this patient?
Evidence of cor pulmonale and a PaO2 between 55 and 60 mm Hg
The PaO2 is less than or equal to 65 mm Hg
Her morning headaches
Dyspnea
All of the above
A man of 38 years old felt ill 2 weeks ago with cough, weakness, increased temperature up to 38,0
°С. His condition suddenly got worse after 7 days of the disease, when chills and sweating appeared,
and evening temperature increased up to 39,0 °С. 2 days prior to hospitalization the patient after
cough expectorated a large amount of stinky bloody sputum; after this he felt better. Objectively:
pulse - 80/min, brteathing rate - 20/min, t - 37,6 °С. What changes are possible on the chest X-ray?
Presence of cavity with horizontal level of fluid
Displacement of mediastinum to the side of homogeneous shade
The homogeneous rounded shade is in the pulmonary tissue
Shade in a lower lobe with diagonal upper border
Darkening of lung lobe
A student complaints on cough with production of mucus-purulent sputum, sometimes with particles
of blood, t - 37,6 °С, weakness, sweating. Since childhood he was often ill with cold, for last several
years there were exacerbations of COPD twice. Doctor suspected bronchiectatic disease in this
patient. What method of investigation may confirm this diagnosis?
Bronchography
Anamnesis morbi
Physical examination of lungs
Scintigraphy of lungs
Tomography of lungs
194.
A. *
B.
C.
D.
E.
195.
A. *
B.
C.
D.
E.
196.
A. *
B.
C.
D.
E.
197.
A. *
B.
C.
D.
E.
198.
A. *
B.
C.
D.
Female patient K., 46 years old, after decreasing of fever after flue noticed pain appeared in a thorax,
cough with yellow-green sputum (amount-150 ml a day), sometimes with some blood. Objectively:
breathing rate - 36/min. In lungs from the right side lower scapula there is dull sound during
percussion, hard breathing, and moist rales. Blood test: L - 18,6х109/l, ESR -64 mm/h. Analysis of
sputum: L -80-100 , Er - 40-50, elastic fibers, cocci. X-ray: lung roots are enlarged, from the right
side lower lobe is heterogeneously infiltrated with two lighter areas. What is the most possible
previous diagnosis?
Right-side
pneumonia with abscesses
Peripheral cancer
Infiltrative tuberculosis in the phase of disintegration
Exudative pleurisy
Infarction-pneumonia
Female, 34 years old, has an increase of body temperature up to 38 °С, cough with purulent sputum,
weakness, dyspnea, pain in a thorax during breathing. During percussion there is shortening of sound
in the lower part of left lung, during auscultation – moist rales. What method of investigation is the
decisive one to confirm diagnosis?
X-ray examination
Bacteriological analysis of sputum
Spirometry
Pneumotachometry
Bronchography
Male patient G., 56 years old, complaints on permanent pain in a thorax which disturbs for the last 2
months. Pain is not connected with breathing. There is also moist cough with particles of blood in
sputum. Weakness, fatigue are present. On the chest X-ray in the lower lobe of right lung there is
spherical shadow, with size of 4x6 cm, related to the lung’s rhadicis. What is the most possible
diagnosis?
Perypheral lung cancer
Tuberculoma
Metastasis
Pulmonary abscess
Pneumonia
Man of 26 years old after measles (in childhood) constantly suffers from cough with production of
mucus-purulent sputum up to 200 ml, mainly in the morning. There is periodical increase of
temperature up to 38 °С. At auscultation - dry, and in lower parts - moist rales. On chest X-ray - there
is “web-like” deformation of pulmonary picture. Your diagnosis?
Bronchiectasis
COPD
Bronchial asthma
Tuberculosis
Pneumosclerosis
Man of 26 years old, complaints on prickly pain during breathing, cough, dyspnea. Objectively: t 37,5 °С, breathing rate - 19/min, heart rate and pulse - 92/min; BP - 120/80. Breathing is vesicular.
From the left side in lateral and lower regions of thorax in the phase of inspiration and expiration
there is sound which increases during pressure with stethoscope and it is preserved after cough. EKG
- without pathological changes. Your diagnosis?
Acute pleurisy
Spontaneous pneumothorax
Intercostal neuralgia
Subcutaneous emphysema
E.
199.
A. *
B.
C.
D.
E.
200.
A. *
B.
C.
D.
E.
201.
A. *
B.
C.
D.
E.
202.
A. *
B.
C.
D.
E.
203.
A. *
B.
C.
D.
Dry pericarditis
Man of 50 y.o. complaints on dyspnea, cough, pain in the left part of thorax, increasing of
temperature up to 37,5 °С. He felt ill three weeks ago. He smokes for 30 years. Objectively: low
feeding, cyanosis of lips, emphysematous thorax, on the left side – from the 6th rib and below a dull
sound is present. Breathing is absent between scapulas from the left side. X-ray: intensive
homogeneous darkening from the left side, shade of heart is shifted to the right. Your diagnosis?
Exudative pleurisy
Athelectasis of lungs
Infiltrative tuberculosis
Dry pleurisy
Pneumonia
Man of 56 years old, alcoholic, felt ill suddenly: temperature increased to 40 °С, weakness, cough
appeared with production of dark sputum. Objectively: condition is severe. T - 39,5 °С. Breathing
rate - 30/min. Heart rate is 100/min., BP - 110/70. In lungs from the right side there are moist rales.
Tones of heart are dull, rhythm is correct, tachycardia. On the chest X-ray film of lungs there is
infiltration of right upper lobe. What complication is the most possible?
Pulmonary abscess
Endocarditis
Bronchiectasis
Pericarditis
Pulmonary bleeding
Man of 54 y.o., complaints on pain in a thorax, dyspnea, cough with bloody sputum. In anamnesis:
long history of cough with purulent sputum up to 200 ml per day, mostly in the morning, periodically
increasing of temperature up to 37,8 °С, sweating, chills. He smokes since 14. Objectively: low
weight, skin with grey tint, edema of face, fingers has shape of "drumsticks", in lungs - pulmonary
and bang-box sound, in some parts dull sound, dry and moist rales. In blood: leucocytosis,
moderately increased ESR. What is the most possible cause of pulmonary bleeding in this case?
Bronchiectasis
Tuberculosis
Chronic bronchitis
Pulmonary abscess
Pulmonary cancer
On the X-ray of 46 y.o. patient, which complaints on increase of temperature and cough with sputum,
in the 10th segment of left lung was founded ring-shape shade with a diameter 8 cm with thick walls
and horizontal level. Other pulmonary parts - without changes. Diagnosis?
Pulmonary abscess
Bronchiectasis
Tubercular cavern
Tuberculoma
Gangrene of lungs
Patient N., 35 y.o., complaints on cough with large amount of purulent sputum in the morning,
sometimes with particles of blood, general weakness, loss of weight. The general condition is
satisfactory, thorax of emphysematous form, breathing rate - 18/min. During auscultation there is
rough breathing, diffuse dry rales. What instrumental method of investigation is the decisive one for
diagnosis?
Bronchography
Chest X-ray
Bronchoscopy with biopsy
Spirometry
E.
204.
A. *
B.
C.
D.
E.
205.
A. *
B.
C.
D.
E.
206.
A. *
B.
C.
D.
E.
207.
A. *
B.
C.
D.
E.
208.
A. *
B.
C.
Scintigraphy
Patient of 19 years old, complaints on increase of temperature up to 39 °С at evenings, severe cough,
production of large amount of sputum with an unpleasant smell. He is ill for several years, the last
exacerbation is related to overcooling. During comparative percussion of lungs – in right lower parts
pulmonary sound is dull, during auscultation – moist rales. What disease is the most possible?
Bronchiectasis
COPD
Abscess of lungs
Gangrene of lungs
Community-acquired pneumonia
Patient of 20 years old complaints on increase of temperature up to 37,5 °С, dyspnea, cough with
purulent sputum, sometimes with particles of blood. Objectively: fingers has shape of "drumsticks".
During auscultation breathing is hard, dry and moist wheezes are present. On X-ray – pulmonary
fibrosis. What is the diagnosis?
Bronchiectasis
Bronchial asthma
COPD
Abscess of lungs
Canceromatosis of lungs
Patient of 25 years old complaints on pain in the left part of thorax, absence of appetite, severe
sweating; after 7-days of permanent fever during an attack of cough he produced 150 ml of yellow
sputum. Objectively: temperature - 38,7 C; breathing rate – 22/min; pulse-96/min; BP 110/70. Chest
X-ray: from the left side there is shade of round form with clear borders, with light areas in a center.
What auscultative sign will confirm the diagnosis of this patient?
Amphoryc breathing
Crepitation
Moist rales
Bronchial breathing
Dry rales
Patient of 28 years old, complaints on severe prickly pain in the left part of thorax, connected with
breathing, dry cough, subfebrile temperature. He is ill for 3 days, after overcooling. Objectively:
position is on the right side, left part of thorax falls behind during breathing, there is clear pulmonary
sound at percussion, vesicular breathing is decreased from the left side, in the phase of inspiration
and expiration there is sound which increases during pressure with stethoscope and it doesn’t
disappear saved after cough. On chest X-ray pulmonary fields are without changes, left sinus is
opened incompletely. The most possible diagnosis:
Dry pleurisy
Pneumonia
Acute bronchitis
Spontaneous pneumothorax
COPD
Patient of 50 years old after overcooling developed a fever up to 40 °С, dyspnea and pain in the right
part of thorax are present. Approximately 100 ml of purulent sputum with bloody particles and
unpleasant smell was producted. On chest X-ray in the right lung there is unhomogeneous massive
infiltration with two bright areas. Mycobacteria tuberculosis and atypical cells in sputum were not
founded. Diagnosis?
Acute abscess of lungs
Gangrene of lungs
Infiltrative tuberculosis with destruction
D.
E.
209.
A. *
B.
C.
D.
E.
210.
A. *
B.
C.
D.
E.
211.
A. *
B.
C.
D.
E.
212.
A. *
B.
C.
D.
E.
213.
A. *
B.
C.
Tumor of lungs with destruction
Empyema of pleura
Patient of 50 years old, entered hospital on the 9th day of disease with complaints on increase of
temperature up to 38,5 °С, acute weakness, pain in the area of right scapula during breathing, dry
cough. Objectively: Breathing rate -28/min. Pulse - 100/min, signs of intoxication. Under the area of
right scapula there is dull percussion sound, bronchial breathing, single moist rales and crepitation.
Three days later there was an attack of cough with production of 200 ml of purulent sputum, after
that temperature of body decreased. On the chest X-ray film under the corner of scapula an
infiltration of lungs of round shape with horizontal level of fluid was found. Diagnosis?
Acute pulmonary abscess
Pulmonary cyst
Cancer of lungs with destruction
Bronchiectasis
Empyema of pleura
Patient came to doctor with exacerbation of COPD. During examination were founded signs of
respiratory insufficiency of II degree. What symptom is the main clinical sign of respiratory
insufficiency of the II degree?
Presence of dyspnea during usual physical exertion
Worsening of external breathing functions
Presence of arterial hypoxemia as cyanosis
Hypertrophy of muscles of neck and abdominal press
Perypheral oedema
Patient complaints on severe cough with production of 600 ml a day purulent sputum of chocolate
color with a putrid smell. Felt ill suddenly, temperature - 39 °С. On the chest X-ray film there is an
area of darkening with cavities in a center, with unclear contours and with the level of fluid. What
disease may be suspected?
Pulmonary gangrene
Tubercular cavern
Pulmonary abscess
Bronchiectasis
Cancer of lungs with destruction
Patient has fever, temperature - 39 °С, there is cough with production of sputum with unpleasant
smell and particles of blood. During auscultation of lungs in right lower lobe there is amphoric
breathing, moist rales. On the chest X-ray film: in the right lower lobe there is a cavity 4 cm in
diameter, with the level of fluid. What is the possible diagnosis?
Pulmonary abscess
Gangrene of lungs
Infiltrative tuberculosis
Cancer of lungs
Pneumonia
Patient M., 39 years old, is ill with attacks of cough with yellow-brown sputum, pain in a right side,
related to the deep breathing, sweating. He is ill for 6 days, after overcooling. Used aspirin.
Objectively: T - 39,6 °С, breathing rate - 26/min., pulse - 110/min., BP -110/70. In lower part of right
lung - moist loud rales. X-ray: in right lower lobe there is massive unhomogeneous infiltration with
lighter areas, sinus is not changed. What complication of disease is the most possible?
Abscesses
Dry pleurisy
Empyema of pleura
D.
E.
214.
A. *
B.
C.
D.
E.
215.
A. *
B.
C.
D.
E.
216.
A. *
B.
C.
D.
E.
217.
A. *
B.
C.
D.
E.
218.
A. *
Spontaneous pneumothorax
Pulmonary athelectasis
Patient N., 31 y.o., complaints on cough with production of mucus-purulent sputum with an
unpleasant smell by “full mouth”, subfebrile temperature, dyspnea, loss of body mass. He is ill since
childhood. Objectively: skin is pale, fingers are changed as "drumsticks", nails - "sentinel glass",
percussion sound is mosaical, breathing is hard, places of moist rales in the places of dull percussion
sound. Your diagnosis:
Bronchiectasis
Viral pneumonia
COPD in the phase of exacerbation
Abscess of lungs
Hypoplasia of lungs
Patient of 54 years old, complaints on shortness of breath during mild physical exertion, cough with
some amount of sputum. Objectively: diffuse cyanosis. Thorax of emphysematous form. In lungs
there is a bit weakened vesicular breathing with prolonged expiration, dry wheezes. BP -140/80.
Pulse - 92/min, rhythmical. Spyrometry: FVC – 36 %, FEV1 – 49 %, FEV1/FVC - 50 %. What is the
type of respiratory insufficiency in this patient?
Mixed
Restrictive
Obstructive
Respiratory insufficiency is not present
It is impossible to make conclusion
Patient of 32 y.o. complaints on dyspnea, pain in the right side of thorax during breathing, cough with
ferruginous sputum, fever with chills, weakness. Objectively: breathing rate - 24/min, vocal fremitus
is increased in the back-lower region of the right part of thorax, during percussion sound is dull, at
auscultation – crepitation is heard. After 5 days of treatment dyspnea increased, vocal fremitus
became weaker, and it is almost impossible to hear vesicular breathing. What complication developed
in this patient?
Exudative pleurisy
Athelectasis
Carnification of lungs
Abscesses
Pneumothorax
Patient of 32 y.o., who drunk a lot of alcohol, has severe pneumonia. His condition was worsened,
the temperature of body rose to 39-40 °С, an unpleasant smell appeared from a mouth, increased
amount of purulent sputum; increased ESR and amount of band leucocytes. On the X-ray - in the
lower lobe of right lung there is massive infiltration with bright area in a center. What complication is
it necessary to suspect?
Acute pulmonary abscess
Bronchiectasis
Infarction-pneumonia
Gangrene of lungs
Empyema of pleura
Patient of 51 y.o. complaints on severe dyspnea, pain in the left part of thorax. Objectively: breathing
rate - 30/min, heart rate - 108/min. Above the left part of thorax vocal fremitus is significantly
decreased, during percussion sound is dull, at auscultation – weak vesicular breathing. On chest
X-ray - homogeneous shade up to the level of the 2nd rib with diagonal upper border, organs of
mediastinum are displaced to the right. Method of choice in the treatment of this patient is:
Pleural punction
B.
C.
D.
E.
219.
A. *
B.
C.
D.
E.
220.
A. *
B.
C.
D.
E.
221.
A. *
B.
C.
D.
E.
222.
A. *
B.
C.
D.
E.
223.
Intravenous usage of large doses of glucocorticosteroids
Emergent bronchoscopy
Inhalation of b2-agonists of short action
Intravenous antibiotics
Patient of 52 y.o. came to the doctor with complaints on temperature 38,6 С, weakness, sweating,
cough with production of purulent sputum (up to 100 ml a day) with an unpleasant smell, pain in the
right part of thorax during breathing. On the chest X-ray it was found “ring-like” shade with the level
of fluid. When abscess may be called “chronic”?
In case of absence of healing signs after a monthly treatment
In case of presence of complications
In the case of development of chronic bronchitis
In the case of development of diffuse pneumosclerosis
In the case of development of pulmonary insufficiency
Patient V. complaints on dyspnea at rest, fever, sweating, pain in a thorax. During examination the
right part of thorax is slowed down in the act of breathing, percussion - dull sound, auscultation absence of respiratory sounds. On the X-ray: homogeneous darkening of 2/3 of right lung. The most
informative diagnostic method in this case:
Pleural punction
Pneumotachometry
Bronchography
Bronchoscopy
Spyrometry
Student of 17 years old, in childhood was often ill with respiratory diseases. In a period between
respiratory viral diseases, cough was present with the production of sputum. Once noticed particles of
blood in sputum. In lungs, especially from the right side, there are different moist rales. Sputum is
mucous-purulent, up to 50 ml per day. On the chest X-ray film – fibrosis and “web” picture mostly in
the right lower lobe. What is the most possible diagnosis?
Bronchiectasis
Chronic pulmonary disease
Chronic bronchitis
Metapneumonic pneumosclerosis
Metatubercular pneumosclerosis
Patient of 36 years old, complaints on dyspnea, feeling of pressure in the right half of thorax,
increasing of temperature up to 38,7 °С, cough with production of small amount of mucus-purulent
sputum. He is ill for a week, after overcooling. Objectively: light acrocyanosis of lips, pulse is
rhythmic, 90/min, BP - 140/85. The right half of thorax is slowed down in the act of breathing.
Percussion – from the right side below corner of scapula there is dull sound. In this region breathing
sounds are not heard. What is the most possible diagnosis?
Right side exudative pleurisy
Bronchiectasis
Right side lower lobe pneumonia
Right side pulmonary athelectasis
Right side abscess of lungs
Patient of 68 years old, complaints on dry cough, elevation of temperature up to 37,5 °С, pain in a
thorax during breathing. 5 days ago he had trauma of thorax. Objectively: pallor, lag of right half of
thorax during breathing. Auscultation – weakened breathing and sound of friction of pleura in right
lower region. In blood: leucocytosis, increased ESR. X-ray - pulmonary fields are not changed. Your
clinical diagnosis?
A. *
B.
C.
D.
E.
224.
A. *
B.
C.
D.
E.
225.
A. *
B.
C.
D.
E.
226.
A. *
B.
C.
D.
E.
227.
A.
B.
C. *
D.
E.
228.
A. *
B.
Dry pleurisy
Pneumonia
Exudative pleurisy
Thraumatic pneumothorax
Cancer of lungs
Patient D., 47 years old, came to the doctor with complaints on fever up to 39 °С, sweating, dry
cough, dyspnea, pain in the right part of thorax during deep breathing and cough. Felt ill suddenly six
days ago. Objectively: condition is severe, skin is pale, breathing rate - 28/min, breathing sounds are
not heard in the lower lobe of right lung, during percussion there is dull sound, during auscultation breathing is significantly weakened. Your previous diagnosis:
Right-side exudative pleurisy
Pneumonia
Acute bronchitis
Thromboembolia of pulmonary artery
Right-side hydrothorax
Patient L., 26 years old, with left-side lower-lobe pneumonia during cough feels acute pain in the left
part of thorax. Objectively: diffuse cyanosis, dilation of the left half of thorax, at percussion tympanitis, auscultation - absence of respiratory sounds above the left part of thorax. Displacement of
right border of heart to the medioclavicular line. What method of investigation will be the most
informative in this case:
Chest X-ray
Bronchoscopy
Bronchography
Pneumotachymetry
Spyrometry
Student of 22, felt ill suddenly. He has fever up to 39 °С, cough, pain under the right scapula during
inspiration and cough. In 3 days the dyspnea increased, respiratory rate is up to 32/min. Below the
corner of right scapula there is dull percussion sound, vocal fremitus is decreased, breathing sounds
are not auscultated. The most effective method of treatment is:
Pleural punction
Prescribing of furosemid
Bed regimen
Prescribing of cephalosporin
Physiotherapy
The patient who has experienced blunt trauma to the chest is at risk for developing a hemothorax.
Which would the physician expect to find in a patient with a hemothorax?
Hemoptysis
Paradoxical chest movements
Percussion dullness on affected side
Hypertympanic sound on affected side
Non of above
Woman of 55 years old, complaints on dull pain in the right half of thorax, unproductive cough,
dyspnea. In anamnesis – rheumatic fever. During examination of lungs from the right side from the
third rib and below there is dull sound at percussion, breathing in this area is significantly weakened.
On the chest X-ray: homogeneous darkening with diagonal upper border from the 3rd rib and below.
Analysis of received exudate: Rivalt test - positive, microscopy revealed lymphocytes. Previous
diagnosis? serous pleurisy
Exudative
Right-side hydrothorax
C.
D.
E.
229.
A. *
B.
C.
D.
E.
Empyema of pleura
Chylothorax
Mezotelioma of pleura
Woman of 58 years old, entered the hospital with complaints on dyspnea and palpitation.
Objectively: condition is severe, nervous, breathing is noisy with participation of additional breathing
muscles, periodical cramps, diffuse cyanosys. In lungs – diffuse dry rales, in lower parts of lungs
breathing is significantly weaker. Pulse - 100/min, oedema, 3 extrasystoles/min, BP - 140/100, Ра O2
- 45, pH - 7,3. What is the main syndrome in this case?
Respiratory insufficiency
Blood hypertension
Tachycardia
Arrhythmia
Heart failure